GI NCLEX Questions

¡Supera tus tareas y exámenes ahora con Quizwiz!

Management of the patient with acute pancreatitis include a. surgery to remove the inflamed pancreas b. pancreatic enzymes administered with meals c. NG suction to prevent gastric contents from entering the duodenum d. endoscopic pancreatic sphncterectomy using ERCP

c. NG suction to prevent gastric contents from entering the duodenum

Patient admitted to ER has profuse bright-red hematemesis. During intial care of the patient, the nurse's first priority is to: a. perform a nursing assessment of patient's status b. establish 2 IV sites c. obtain a thorough health history d. perform a gastric lavage with cool tap water in prep for endoscopic exam

A

The client being seen in a physician's office has just been scheduled for a barium swallow the next day. The nurse writes down which of the following instructions for the client to follow before the test? A. Fast for 8 hours before the test B. Eat a regular supper and breakfast C. Continue to take all oral medications as scheduled D. Monitor own bowel movement pattern for constipation

A A barium swallow is an x-ray study that uses a substance called barium for contrast to highlight abnormalities in the GI tract. The client should fast for 8 to 12 hours before the test, depending on the physician instructions. Most oral medications also are withheld before the test. After the procedure the nurse must monitor for constipation, which can occur as a result of the presence of barium in the GI tract.

After a subtotal gastrectomy, the nurse should anticipate that nasogastric tube drainage will be what color for about 12 to 24 hours after surgery? A. Dark brown B. Bile green C. Bright red D. Cloudy white

A About 12 to 24 hours after a subtotal gastrectomy, gastric drainage is normally brown, which indicates digested blood. Bile green or cloudy white drainage is not expected during the first 12 to 24 hours after a subtotal gastrectomy. Drainage during the first 6 to 12 hours contains some bright red blood, but large amounts of blood or excessively bloody drainage should be reported to the physician promptly.

A client has just had surgery for colon cancer. Which of the following disorders might the client develop? A. Peritonitis B. Diverticulosis C. Partial bowel obstruction D. Complete bowel obstruction

A Bowel spillage could occur during surgery, resulting in peritonitis. Complete or partial bowel obstruction may occur before bowel resection. Diverticulosis doesn't result from surgery or colon cancer.

The nurse provides medication instructions to a client with peptic ulcer disease. Which statement, if made by the client, indicates the best understanding of the medication therapy? A. "The cimetidine (Tagamet) will cause me to produce less stomach acid." B. "Sucralfate (Carafate) will change the fluid in my stomach." C. "Antacids will coat my stomach." D. "Omeprazole (Prilosec) will coat the ulcer and help it heal."

A Cimetidine (Tagamet), a histamine H2 receptor antagonist, will decrease the secretion of gastric acid. Sucralfate (Carafate) promotes healing by coating the ulcer. Antacids neutralize acid in the stomach. Omeprazole (Prilosec) inhibits gastric acid secretion.

Which of the following dietary measures would be useful in preventing esophageal reflux? A. Eating small, frequent meals B. Increasing fluid intake C. Avoiding air swallowing with meals D. Adding a bedtime snack to the dietary plan

A Esophageal reflux worsens when the stomach is overdistended with food. Therefore, an important measure is to eat small, frequent meals. Fluid intake should be decreased during meals to reduce abdominal distention. Avoiding air swallowing does not prevent esophageal reflux. Food intake in the evening should be strictly limited to reduce the incidence of nighttime reflux, so bedtime snacks are not recommended.

Which of the following areas is the most common site of fistulas in client's with Crohn's disease? A. Anorectal B. Ileum C. Rectovaginal D. Transverse colon

A Fistulas occur in all these areas, but the anorectal area is most common because of the relative thinness of the intestinal wall in this area.

Dr. Smith has determined that the client with hepatitis has contracted the infection from contaminated food. The nurse understands that this client is most likely experiencing what type of hepatitis? A. Hepatitis A B. Hepatitis B C. Hepatitis C D. Hepatitis D

A Hepatitis A is transmitted by the fecal-oral route via contaminated food or infected food handlers. Hepatitis B, C, and D are transmitted most commonly via infected blood or body fluids.

Histamine2-receptor antagonists: A. Compete with histamine for binding sites on the parietal cells B. Irreversibly bind to H+/K+ATPase C. Cause a decrease in stomach pH D. Decrease signs and symptoms of allergies related to histamine release

A Histamine receptor blocking agents decrease gastric acid by competing with histamine for binding sites on the parietal cells.

Which assessment finding indicates that lactulose is effective in decreasing the ammonia level in the client with hepatic encephalopathy? A. Passage of two or three soft stools daily B. Evidence of watery diarrhea C. Daily deterioration in the client's handwriting D. Appearance of frothy, foul-smelling stools

A Lactulose reduces serum ammonia levels by inducing catharsis, subsequently decreasing colonic pH and inhibiting fecal flora from producing ammonia from urea. Ammonia is removed with the stool. Two or three soft stools daily indicate effectiveness of the drug. Watery diarrhea indicates overdose. Daily deterioration in the client's handwriting indicates an increase in the ammonia level and worsening of hepatic encephalopathy. Frothy, foul-smelling stools indicate steatorrhea, caused by impaired fat digestion.

Radiation therapy is used to treat colon cancer before surgery for which of the following reasons? A. Reducing the size of the tumor B. Eliminating the malignant cells C. Curing the cancer D. Helping the bowel heal after surgery

A Radiation therapy is used to treat colon cancer before surgery to reduce the size of the tumor, making it easier to be resected. Radiation therapy isn't curative, can't eliminate the malignant cells (though it helps define tumor margins), can could slow postoperative healing.

The client has been admitted with a diagnosis of acute pancreatitis. The nurse would assess this client for pain that is: A. Severe and unrelenting, located in the epigastric area and radiating to the back. B. Severe and unrelenting, located in the left lower quadrant and radiating to the groin. C. Burning and aching, located in the epigastric area and radiating to the umbilicus. D. Burning and aching, located in the left lower quadrant and radiating to the hip.

A The pain associated with acute pancreatitis is often severe and unrelenting, is located in the epigastric region, and radiates to the back.

A 40-year-old male client has been hospitalized with peptic ulcer disease. He is being treated with a histamine receptor antagonist (cimetidine), antacids, and diet. The nurse doing discharge planning will teach him that the action of cimetidine is to: A. Reduce gastric acid output B. Protect the ulcer surface C. Inhibit the production of hydrochloric acid (HCl) D. Inhibit vagus nerve stimulation

A These drugs inhibit action of histamine on the H2 receptors of parietal cells, thus reducing gastric acid output.

A patient with chronic alcohol abuse is admitted with liver failure. You closely monitor the patient's blood pressure because of which change that is associated with the liver failure? A. Hypoalbuminemia B. Increased capillary permeability C. Abnormal peripheral vasodilation D. Excess rennin release from the kidneys

A Blood pressure decreases as the body is unable to maintain normal oncotic pressure with liver failure, so patients with liver failure require close blood pressure monitoring. Increased capillary permeability, abnormal peripheral vasodilation, and excess rennin released from the kidney's aren't direct ramifications of liver failure.

Most effective means of suppressing pancreatic secreation during an episode of pancreatitis is the use of: a. antibiotics b. NPO status c. antispasmotics d. H2R blockers

B

The client with a colostomy has an order for irrigation of the colostomy. The nurse used which solution for irrigation? A. Distilled water B. Tap water C. Sterile water D. Lactated Ringer's

B Warm tap water or saline solution is used to irrigate a colostomy. If the tap water is not suitable for drinking, then bottled water should be used.

A patient is returned to the surgical unit following a laparoscopic fundoplication for repair of hiatal hernia with an IV, NG tube t osuction, and several small abdominal incisions. to prevent disruption of the surgical site, it is most important for the nurse to: a. monitor for return of perstalsis b. position the patient on the right side c. maintain the patency of the NG tube d. assess abdominal wounds

C

Combined with clinical manifestations, the lab finding that is most commonly used to diagnose acute prancreatitis is: a. increased serum lipase c c. increased urinary amylase d. decreased renal amylase creatine clearance

C

Patient with history of PUD is hospitalized with symptoms of a perforation. During initial assessment nurse would expect to find: a. vomit of bright red blood b. projectile vomiting c. sudden, severe upper abdominal pain and shoulder pain d. hyperactive stomach sounds

C

When obtaining a nursing history on a client with a suspected gastric ulcer, which signs and symptoms would the nurse expect to see? Select all that apply. A. Epigastric pain at night B. Relief of epigastric pain after eating C. Vomiting D. Weight loss

C,D Vomiting and weight loss are common with gastric ulcers. Clients with a gastric ulcer are most likely to complain of a burning epigastric pain that occurs about one hour after eating. Eating frequently aggravates the pain. Clients with duodenal ulcers are more likely to complain about pain that occurs during the night and is frequently relieved by eating.

Bismuth subsalicylate (Pepto-Bismol), as an absorbent, has which of the following mechanisms of action? A. Decreased GI motility B. Decreased gastric secretions C. Increased fluid absorption D. Binding to diarrhea-causing bacteria for excretion

D Absorbent antidiarrheal medications bind to diarrhea-causing bacteria to form a nonabsorbable complex, which is then excreted in the stool.

Which of the following therapies is not included in the medical management of a client with peritonitis? A. Broad-spectrum antibiotics B. Electrolyte replacement C. I.V. fluids D. Regular diet

D The client with peritonitis usually isn't allowed anything orally until the source of peritonitis is confirmed and treated. The client also requires broad-spectrum antibiotics to combat the infection. I.V. fluids are given to maintain hydration and hemodynamic stability and to replace electrolytes.

Which nursing intervention is essential immediately following a gastroduodenoscopy? 1. Force fluids. 2. Position him supine. 3. Instruct him not to eat or drink. 4. Encourage coughing and deep breathing

(3) It is essential to keep him NPO until the cough and gag reflexes have returned. He should be in a semi-Fowler's position to reduce edema formation

A patient diagnosed with inflammatory bowel disease experiences an obstruction in the small bowel. When assessing the patient, which of the following will the healthcare provider anticipate? Please choose from one of the following options. A. Scaphoid abdomen B. Hypovolemia C. Increased flatus D. Passage of melena

B

Which of the following terms best describes the pain associated with appendicitis A. Aching B. Fleeting C. Intermittent D. Steady

D The pain begins in the epigastrium or periumbilical region, then shifts to the right lower quadrant and becomes steady. The pain may be moderate to severe.

Combined with clinical manifestations, the laboratory finding that is most commonly used to diagnose acute pancreatitis is a. increased serum calcium b. increased serum amylase c. increased urinary amylase d. decreased serum glucose

b. increased serum amylase

An adult who has cholecystitis reports clay colored stools and moderate jaundice. Which is the best explanation for the presence of clay colored stools and jaundice? 1. There is an obstruction in the pancreatic duct. 2. There are gallstones in the gallbladder. 3. Bile is no longer produced by the gallbladder. 4. There is an obstruction in the common bile duct.

4) Clay colored stools means bile is not getting through to the duodenum. The bile duct is obstructed so bile backs up into the bloodstream causing jaundice

he nurse is caring for a client who has had a colostomy.Which of the following client behaviors is indicative of a willingness to be involved in self-care following a colostomy? 1. Discussing the cost of his hospitalization. 2. Asking what time the surgeon will be in. 3. Asking questions about the equipment being used. 4. Complaining about the noise in the adjacent room

3) When the client asks questions about theequipment being used, he indicates a readiness to learn.None of the other responses indicate a willingness tolearn about his colostom

Atropine 0.5 mg is ordered for a client having an acute attack of cholecystitis. What is the primary purpose of this drug for this client? To 1. decrease skeletal muscle spasms. 2. increase gastrointestinal peristalsis 3. decrease smooth muscle contractions 4. decrease anxiety

3)Atropine is an anticholinergic drug , which will decrease contractions of the gallbladder

Which of the following nursing interventions should the nurse perform for a female client receiving enteral feedings through a gastrostomy tube? a. Change the tube feeding solutions and tubing at least every 24 hours. b. Maintain the head of the bed at a 15-degree elevation continuously. c. Check the gastrostomy tube for position every 2 days. d. Maintain the client on bed rest during the feedings.

A male client with pancreatitis complains of pain. The nurse expects the physician to prescribe meperidine (Demerol) instead of morphine to relieve pain because: a. meperidine provides a better, more prolonged analgesic effect. b. morphine may cause spasms of Oddi's sphincter. c. meperidine is less addictive than morphine. d. morphine may cause hepatic dysfunction.

Mr. Hasakusa is in end-stage liver failure. Which interventions should the nurse implement when addressing hepatic encephalopathy? (Select all that apply.) A. Assessing the client's neurologic status every 2 hours B. Monitoring the client's hemoglobin and hematocrit levels C. Evaluating the client's serum ammonia level D. Monitoring the client's handwriting daily E. Preparing to insert an esophageal tamponade tube F. Making sure the client's fingernails are short

A,C,D Hepatic encephalopathy results from an increased ammonia level due to the liver's inability to covert ammonia to urea, which leads to neurologic dysfunction and possible brain damage. The nurse should monitor the client's neurologic status, serum ammonia level, and handwriting. Monitoring the client's hemoglobin and hematocrit levels and insertion of an esophageal tamponade tube address esophageal bleeding. Keeping fingernails short address jaundice.

The nurse caring for a client with small-bowel obstruction would plan to implement which nursing intervention first? a. Administering pain medication b. Obtaining a blood sample for laboratory studies c. Preparing to insert a nasogastric (NG) tube d. Administering I.V. fluids

Answer D. I.V. infusions containing normal saline solution and potassium should be given first to maintain fluid and electrolyte balance. For the client's comfort and to assist in bowel decompression, the nurse should prepare to insert an NG tube next. A blood sample is then obtained for laboratory studies to aid in the diagnosis of bowel obstruction and guide treatment. Blood studies usually include a complete blood count, serum electrolyte levels, and blood urea nitrogen level. Pain medication often is withheld until obstruction is diagnosed because analgesics can decrease intestinal motility.

Patient asks nurse if his risks for colon cancer are increased due to a polyp. the best response is: a. it is very rare for polyps ot become malignant b. individuals with polyps have a 100% lifetime risk of developing colorectal cancer c. all polyps are abnormal and should be removed, but the risk for cancer depends on the type and if malignant changes are present d. all polyps are premalignant and source of most colon cancer. get colonoscpy q 6 months.

C

In discussing long term management with the patient with alcoholic cirrhosis, the nurse advises the patient that a. a daily exercise regimen is important to increase the blood flow through the liver b. cirrhosis can be reversed if the patient follows a regimen of proper rest and nutrition c. abstinence from alcohol is the most important factor in improvement of the patient's condition d. the only over the counter analgesic that should be used for minor aches and pains is acetaminophen

C- Abstinence from alcohol is very important in alcoholic cirrhosis and may result in improvement if started when liver damage is reduced by rest and nutrition, most changes in the liver cannot be reversed. Exercise does not promote portal circulation, and very moderate exercise is recommended. Acetaminophen should not be used by the patient with liver disease because it is potentially hepatotoxic.

A patient who has been vomiting for several dasy from an unknown cause is admitted to hospital. the nurse anticipates collaborative care to indlude: a. oral admin of broth and tea b. admin of paretneral antiemetics c. insertion of NG tube to suction d. IV replacement of fluid and electrolytes

D

A patient with cirrhosis that is refractory to other treatments for esophageal varices undergoes a peritoneovenous shunt. As a result of this procedure, the nurse would expect the patient to experience a. an improved survival rate b. decreased serum ammonia levels c. improved metabolism of nutrients d. improved hemodynamic function and renal perfusion

D- By shunting fluid sequestered in the peritoneum into the venous system, pressur eon esophageal veins is decreased, and more volume is returned to the circulation, improving CO and renal perfusion. However, because ammonia is diverted past the liver, hepatic encephalopathy continues. These procedures do not prolong life or promote liver function.

The nurse recognizes early signs of hepatic encephalopathy in the patient who a. manifests asterixis b. becomes unconscious c. has increasing oliguria d. is irritable and lethargic

D- Early signs of this neurologic condition include euphoria, depression, apathy, irritability, confusion, agitation, drowsiness, and lethargy. Loss of consciousness is usually preceded by asterixis, disorientation, hyperventilation, hypothermia, and alterations in reflexes. Increasing oliguria is a sign of hepatorenal syndrome.

The family members of a patient with hepatitis A ask if there is anything that will prevent them from developing the disease. The best response by the nurse is a. "no immunization is available for hepatitis A, nor are you likely to get the disease" b. "only individuals who have had sexual contact with the patient should receive immunization" c. "all family members should receive the hepatitis A vaccine to prevent or modify the infection" d. "those who have had household or close contact with the patient should receive immune globulin"

D- Individuals who have been exposed to hepatitis A through household contact or foodborne outbreaks should be given immune globulin within 1 to 2 weeks of exposure to prevent or modify the illness. Hep A vaccine is used to provide pre-exposure immunity to the virus and is indicated for individuals at high risk for hep A exposure. Although hep A can be spread by sexual contact, the risk is higher for transmission with the oral-fecal route.

A 74 y.o. female pt w/ GERD takes over-the-counter meds. For which med, if taken long-term, should the nurse teach about increased risk of fractures? Sucralfate (Carafate) Cimetidine (Tagamet) Omeprazole (Prilosec) Metoclopramide (Reglan)

Omeprazole (Prilosec) There is a potential link between proton pump inhibitor use and bone metabolism. Long-term use or high doese of these may increase the risk of fractures of the hip, wrist, and spine. Lower doses or shorter duration of therapy should be considered.

The patient with chronic pancreatitis is more likely than the patient with acute pancreatitis to a. need to abstain from alcohol b. experience acute abdominal pain c. have malabsorption and diabetes mellitus d. require a high carbohydrate, high protein, low fat diet

c. have malabsorption and diabetes mellitus Chronic damage to the pancreas causes pancreatic exocrine and endocrine insufficiency, resulting in a deficiency of digestive enzymes and insulin. Malabsorption and diabetes often result

The nurse determines that further discharge instruction is needed when the patient with acure pancreatitis states, a. i should observe for fat in my stools b. i must not use alcohol to prevent future attacks of pancreatitis c. i shouldn't eat salty foods or foods with high amounts of sodium d. i will need to continue to monitor my blood glucose levels until my pancreas is healed

c. i shouldn't eat salty foods or foods with high amounts of sodium Sodium restriction isn't indicated for pancreatitis

An adult male client is admitted with a diagnosis of probable duodenal ulcer. Which of the following laboratory tests would it be most essential for the nurse to assess immediately? 1. Hemoglobin and Hematocrit 2. SGPT and SGOT 3. Na and K 4. BUN and creatinine

(1)Hgb and Hct would indicate if there had been any bleeding from the ulcer. SGPT and SGOT elevations indicate liver damage. Na and K indicate electrolyte imbalances. BUN and creatinine elevationswould indicate renal disease.

A barium enema is ordered for an adult male client. The nurse is teaching him what to expect regarding the procedure. Which statement should be included in the teaching? 1. Fecal matter must be cleansed from the bowel for good visualization. 2. There will be no food restrictions before the test .3. He will not have to change positions during the procedure. 4. He will be asked to drink barium during the procedure

(1) The bowel must be free of fecal material for good visualization of the bowel. He will be on a clear liquid or low residue diet for the day preceding the exam. The client is put in several positions during the test. Barium is given by enema. It is given by mouth in an upper GI series

An adult client is to have a sigmoidoscopy in the morning.What should the nurse plan to do? 1. Give him an enema 1 hour before the examination. 2. Keep him NPO for 8 hours before the examination. 3. Order a low fat, low residue diet for breakfast. 4. Administer enemas until clear this evening

(1) An enema 1 hour before the exam will clear the sigmoid colon. A client having an upper GI series will be NPO. Low fat diet is indicated prior to a gallbladder series. Low residue diet is part of the preparation for barium enema. Enemas until clear are sometime sordered prior to a barium enema

The nurse is preparing a client with Crohn's disease for discharge. Which statement he makes indicates he needs further teaching? 1. "Stress can make it worse." 2. "Since I have Crohn's disease I don't have to worry about colon cancer." 3. "I realize I shall always have to monitor my diet." 4. "I understand there is a high incidence of familial occurrence with this disease

(2) Persons with Crohn's disease are at high risk for the development of colon cancer. The other answers are all correct

The client asks how he contracted hepatitis A. He reports all of the following. Which one is most likely related to hepatitis A? 1. He ate home canned tomatoes. 2. He ate oysters his roommate brought home from a fishing trip. 3. He stepped on a nail 2 weeks ago. 4. He donated blood 2 weeks before he got sick

(2) Shellfish that grow in contaminated waters may have the virus. Home canned tomatoes might cause food poisoning. Stepping on a nail might cause tetanus.Donating blood will not cause hepatitis. Receiving blood might cause hepatitis B or C

An adult client is to have a gastroduodenoscopy in the morning.The nurse's instructions should include the information that he will be 1. given a general anesthetic during the procedure. 2. given a local anesthetic to ease the discomfort during the procedure. 3. asked to assist by coughing during the procedure. 4. asked to assist by performing a Valsalva maneuver during the procedure

(2)Gastroduodenoscopy is visualization of the esophagus, stomach and duodenal through a flexible tube inserted orally. The exam is uncomfortable because the muscles of the GI tract have spasms as the tube is passed. This causes difficulty swallowing. The client is usually given a local anesthetic to the posterior pharynx to reduce the discomfort during the passage of the tube.He may also be given conscious sedation. He will not given a general anesthetic because he must be able to assist by swallowing. Coughing and the performance of a Valsalva maneuver would impede the passage of the tube.

In preparation for an abdomino-perineal resection the client is placed on a low residue diet. Which of the following food lists is appropriate for him to eat on a low residue diet? 1. Ground lean beef, soft boiled eggs, tea. 2. Lettuce, spinach, corn. 3. Prunes, grapes, apples. 4. Bran cereal, whole wheat toast, coffee

1)All of these foods are low in residue. Fruits,vegetables and whole grains are high in residue.

A patient diagnosed with viral hepatitis is in the pre-icteric phase. When assessing the patient, which of these findings should the healthcare provider anticipate? Select all that apply. Select all that apply. 1. Nausea 2. Pruritis 3. Tarry stools 4. Anorexia 5. Dark urine

1, 4

A nurse is assessing a client who has been admitted with a diagnosis of an obstruction in the small intestine. The nurse should assess the client for? Select all that apply. 1. Projectile vomiting. 2. Significant abdominal distention. 3. Copious diarrhea. 4. Rapid onset of dehydration. 5. Increased bowel sounds.

1, 4, 5. Signs and symptoms of intestinal obstructions in the small intestine may include projectile vomiting and rapidly developing dehydration and electrolyte imbalances. The client will also have increased bowel sounds, usually high-pitched and tinkling. The client would not normally have diarrhea and would have minimal abdominal distention. Pain is intermittent, being relieved by vomiting. Intestinal obstructions in the large intestine usually evolve slowly, produce persistent pain, and vomiting is less common. Clients with a large-intestine obstruction may develop constipation and significant abdominal distention.

A client is admitted with a bowel obstruction. The client has nausea, vomiting, and crampy abdominal pain. The physician has written orders for the client to be up ad lib, to have narcotics for pain, to have a nasogastric tube inserted if needed, and for I.V. Ringer's Lactate and hyperalimentation fluids. The nurse should do the following in order of priority from first to last: 1. Assist with ambulation to promote peristalsis 2. Administer Ringer's Lactate 3. Insert a nasogastric tube. 4. Start and infusion of hyperalimentation fluids

1,2,3,4 The nurse should first help the client ambulate to try to induce peristalsis; this may be effective and require the least amount of invasive procedures. I.V. fluid therapy can be done to correct fluid and electrolyte imbalances (sodium and potassium), and normal saline or Ringer's Lactate to correct interstitial fluid deficit. Nasogastric (NG) decompression of G.I. tract to reduce gastric secretions and nasointestinal tubes may also be used. Hyperalimentation can be used to correct protein deficiency from chronic obstruction, paralytic ileus, or infection.

The physician orders intestinal decompression with a Cantor tube for a client with an intestinal obstruction. In order to determine effectiveness of intestinal decompression the nurse should evaluate the client to determine if: 1. Fluid and gas have been removed from the intestine. 2. The client has had a bowel movement. 3. The client's urinary output is adequate. 4. The client can sit up without pain.

1. Intestinal decompression is accomplished with a Cantor, Harris, or Miller-Abbott tube. These 6- to 10-foot tubes are passed into the small intestine to the obstruction. They remove accumulated fluid and gas, relieving the pressure. The client will not have an adequate bowel movement until the obstruction is removed. The pressure from the distended intestine should not obstruct urinary output. While the client may be able to more easily sit up, and the pain caused by the intestinal pressure will be less, these are not the primary indicators for successful intestinal decompression.

A client is admitted to the hospital with ulcerative colitis.Admitting orders include a low residue diet. Which food would be contraindicated for this client? . Roast beef. 2. Fresh peas .3. Mashed potatoes .4. Baked chicken

2) Fresh peas are high in residue. The other foods are low in residue

The client with a duodenal ulcer is ready for discharge. Which statement made by the client indicates a need for more teaching about his diet? 1. "It's a good thing I gave up drinking alcohol last year." 2. "I will have to drink lots of milk and cream every day." 3. "I will stay away from cola drinks after I am discharged." 4. "Eating three nutritious meals and snacks every day is okay.

2) Milk and cream are now known to cause rebound acidity and are not prescribed for ulcer clients.The other choices all indicate good knowledge. He should not drink alcohol or cola. Three meals and snacks will help to keep the stomach from staying empty for long periods

Which of the following statements about nasoenteric tubes is correct? 1. The tube cannot be attached to suction. 2. The tube contains a soft rubber bag filled with mercury. 3. The tube is taped securely to the client's cheek after insertion. 4. The tube can have its placement determined only by auscultation

2. A nasoenteric tube has a small balloon at its tip that is weighted with mercury. The weight of the mercury helps advance the tube by gravity through the intestine. Nasoenteric tubes are attached to suction. A nasoenteric tube is not taped in position until it has reached the obstruction. Because the tube has a radiopaque strip, its progress through the intestinal tract can be followed by fluoroscopy.

After insertion of a nasoenteric tube, the nurse should place the client in which position? 1. Supine. 2. Right side-lying. 3. Semi-Fowler's. 4. Upright in a bedside chair.

2. The client is placed in a right side-lying position to facilitate movement of the mercury-weighted tube through the pyloric sphincter. After the tube is in the intestine, the client is turned from side to side or encouraged to ambulate to facilitate tube movement through the intestinal loops. Placing the client in the supine or semi-Fowler's position, or having the client sitting out of bed in a chair will not facilitate tube progression

The client with an intestinal obstruction continues to have acute pain even though the nasoenteric tube is patent and draining. Which action by the nurse would be most appropriate? 1. Reassure the client that the nasoenteric tube is functioning. 2. Assess the client for a rigid abdomen. 3. Administer an opioid as ordered. 4. Reposition the client on the left side.

2. The client's pain may be indicative of peritonitis, and the nurse should assess for signs and symptoms, such as a rigid abdomen, elevated temperature, and increasing pain. Reassuring the client is important, but accurate assessment of the client is essential. The full assessment should occur before pain relief measures are employed. Repositioning the client to the left side will not resolve the pain.

Because a client has a nasogastric tube attached to intermittent drainage the nurse should be particularly alert for the development of which complication? 1. Hypocalcemia. 2. Hypermagnesemia. 3. Hypokalemia. 4. Hypoglycemia

3) Potassium is present in GI fluids and is lost during suctioning.

An adult is admitted with a duodenal ulcer. On the second day after admission, the client develops severe, persistent pain radiating to the shoulder. What action should the nurse take first? 1. Notify the physician. 2. Place client in a high-Fowler's position to decrease pressure on the gastric area and shoulder. 3. Examine the client for board-like rigidity of the abdomen. 4. Administer ordered prn pain medication

3) The nurse should first do a quick assessment todetermine if the cause of the pain is more apt to beperforation of the ulcer or something else such as cardiacpain. If the ulcer has perforated the client's abdomen willbe tender and rigid - board like

The client with appendicitis asks the nurse for a laxative to help relieve her constipation. The nurse explains to her that laxatives are not given to persons with possible appendicitis. What is the primary reason for this? 1. Laxatives will decrease the spread of infection. 2. Laxatives are not given prior to any type of surgery. 3. The patient does not have true constipation. She only has pressure. 4. Laxatives could cause rupture of the appendix.

4) Laxatives cause increased peristalsis, which may cause the appendix to rupture. #2 is not a true statement. Laxatives may well be given prior to gynecological, rectal and colon surgery. #3 is true but is not the primary reason why laxatives are not given

A client has an order for irrigation of a nasogastric tube. What should the nurse do before irrigating the nasogastric tube? 1. Inject a small amount of air while listening with a stethoscope over the stomach for a "swoosh." 2. Instill 5 cc of normal saline and observe for development of coughing and dyspnea. 3. Place the end of the nasogastric tube in a glass of water and observe for bubbles. 4. Aspirate and check the pH

4) To determine if the tube is in the stomach, the nurse should aspirate and check the pH. It should be less than 5. Never instill saline. If the tube were in the-bronchi instead of the stomach, saline would cause respiratory distress. Placing the end of the tube in a glass

Following a cholecystectomy, drainage form the T tube for the first 24 hours postoperative was 350 cc. Proper nursing action in response to this should be to 1. notify the physician . 2. raise the level of the drainage bag to decrease rate of flow. 3. increase the IV flow rate to compensate for the loss. 4. continue to observe and measure drainage

4)350 cc in 24 hours after surgery is a normal amount of bile drainage

An adult male is admitted to the hospital complaining of burning epigastric pain. He reports to the nurse that he has gained 14 pounds over the last two months. Which nursing response is best? 1. "Why were you eating more?" 2. "Has the weight gain been intentional?" 3. "Does your weight usually fluctuate this much?" 4. "How did your eating habits change?"

4)Weight gain may occur due to increased consumption of food as the client tries to feed a duodenal ulcer. "Why" questions are threatening to clients. #3asks for a yes or no answer. This will not give as much information as asking about the eating habits

Before abdominal surgery for an intestinal obstruction, the nurse monitors the client's urine output and finds that the total output for the past 2 hours was 35 mL. The nurse then assesses the client's total intake and output over the last 24 hours and notes that he had 2,000 mL of I.V. fluid for intake, 500 mL of drainage from the nasogastric tube, and 700 mL of urine for a total output of 1,200 mL. This would indicate which of the following? 1. Decreased renal function. 2. Inadequate pain relief. 3. Extension of the obstruction. 4. Inadequate fluid replacement.

4. Considering that there is usually 1 L of insensible fluid loss, this client's output exceeds his intake (intake, 2,000 mL; output, 2,200 mL), indicating deficient fluid volume. The kidneys are concentrating urine in response to low circulating volume, as evidenced by a urine output of less than 30 mL/ hour. This indicates that increased fluid replacement is needed. Decreasing urine output can be a sign of decreased renal function, but the data provided suggest that the client is dehydrated. Pain does not affect urine output. There are no data to suggest that the obstruction has worsened.

A patient with NG tube develops nausea and increased upper abominal bowel sounds. Appropriate action is to: a. check the patency of the NG tube b. place client in recumbant position c. assess vital signs d. ecourage deep breathing

A

A patient's vomitus is dark brown and has a coffee-ground appearance. the nurse recognizes that this emsis is charactristic of: a. stomach bleeding b. an intestinal obstruction c. bile reflux d. active bleeding of lower esophagus

A

Management of patient with upper GI bleeding is effective the lab results reveal: a. decreasing BUN b. normal hematocrit c. urine output of 20 ml hr d. specific gravity of 1.03

A

Patient with inflammatory bwel disease has a nursing diagnosis of imbalanced nutrition: less than body requirements r/t decreased nutritional intake and decreased intestinal absorption. Data to support this is: a. pallor and hair loss b. frequent diarrhea stools c. anorectal excoriation and pain d. hypotension and urine output below 30 ml /hr

A

Postop patient has nursing diagnosis of pain r/t to immobility, meds, and decreased motility as evidneced by abdominal pain and distention and inability to pass flatus. An apropriate nursing intervention for the patient is to: a. ambulate patient more frequently b. assess abdomen for bowel sounds c. place patient in high fowlers d. withhold narcotics because they decrease bowel motility

A

A female client being seen in a physician's office has just been scheduled for a barium swallow the next day. The nurse writes down which instruction for the client to follow before the test? A. Fast for 8 hours before the test B. Eat a regular supper and breakfast C. Continue to take all oral medications as scheduled D. Monitor own bowel movement pattern for constipation

A A barium swallow is an x-ray study that uses a substance called barium for contrast to highlight abnormalities in the gastrointestinal tract. The client should fast for 8 to 12 hours before the test, depending on physician instructions. Most oral medications also are withheld before the test. After the procedure, the nurse must monitor for constipation, which can occur as a result of the presence of barium in the gastrointestinal tract.

Which of the following complications is thought to be the most common cause of appendicitis? A. A fecalith B. Bowel kinking C. Internal bowel occlusion D. Abdominal bowel swelling

A A fecalith is a fecal calculus, or stone, that occludes the lumen of the appendix and is the most common cause of appendicitis. Bowel wall swelling, kinking of the appendix, and external occlusion, not internal occlusion, of the bowel by adhesions can also be causes of appendicitis.

Which of the following diets is most commonly associated with colon cancer? A. Low-fiber, high fat B. Low-fat, high-fiber C. Low-protein, high-carbohydrate D. Low carbohydrate, high protein

A A low-fiber, high-fat diet reduced motility and increases the chance of constipation. The metabolic end products of this type of diet are carcinogenic. A low-fat, high-fiber diet is recommended to prevent colon cancer.

The nurse would teach patients that antacids are effective in treatment of hyperacidity because they: A. Neutralize gastric acid B. Decrease stomach motility C. Decrease gastric pH D. Decrease duodenal pH

A Antacids work by neutralizing gastric acid, which would cause an increase in pH. They do not affect gastric motility.

The nurse is reviewing the record of a female client with Crohn's disease. Which stool characteristics should the nurse expect to note documented in the client's record? A. Diarrhea B. Chronic constipation C. Constipation alternating with diarrhea D. Stools constantly oozing from the rectum

A Crohn's disease is characterized by nonbloody diarrhea of usually not more than four to five stools daily. Over time, the diarrhea episodes increase in frequency, duration, and severity. Options B, C, and D are not characteristics of Crohn's disease.

Which of the following symptoms indicated diverticulosis? A. No symptoms exist B. Change in bowel habits C. Anorexia with low-grade fever D. Episodic, dull, or steady midabdominal pain

A Diverticulosis is an asymptomatic condition. The other choices are signs and symptoms of diverticulitis.

The nurse is monitoring a female client for the early signs and symptoms of dumping syndrome. Which of the following indicate this occurrence? A. Sweating and pallor B. Bradycardia and indigestion C. Double vision and chest pain D. Abdominal cramping and pain

A Early manifestations of dumping syndrome occur 5 to 30 minutes after eating. Symptoms include vertigo, tachycardia, syncope, sweating, pallor, palpitations, and the desire to lie down.

Which of the following types of diets is implicated in the development of diverticulosis? A. Low-fiber diet B. High-fiber diet C. High-protein diet D. Low-carbohydrate diet

A Low-fiber diets have been implicated in the development of diverticula because these diets decrease the bulk in the stool and predispose the person to the development of constipation. A high-fiber diet is recommended to help prevent diverticulosis. A high-protein or low-carbohydrate diet has no effect on the development of diverticulosis.

The client has orders for a nasogastric (NG) tube insertion. During the procedure, instructions that will assist in the insertion would be: A. Instruct the client to tilt his head back for insertion in the nostril, then flex his neck for the final insertion B. After insertion into the nostril, instruct the client to extend his neck C. Introduce the tube with the client's head tilted back, then instruct him to keep his head upright for final insertion D. Instruct the client to hold his chin down, then back for insertion of the tube

A NG insertion technique is to have the client first tilt his head back for insertion into the nostril, then to flex his neck forward and swallow. Extension of the neck (2) will impede NG tube insertion.

Which of the following aspects is the priority focus of nursing management for a client with peritonitis? A. Fluid and electrolyte balance B. Gastric irrigation C. Pain management D. Psychosocial issues

A Peritonitis can advance to shock and circulatory failure, so fluid and electrolyte balance is the priority focus of nursing management. Gastric irrigation may be needed periodically to ensure patency of the nasogastric tube. Although pain management is important for comfort and psychosocial care will address concerns such as anxiety, focusing on fluid and electrolyte imbalance will maintain hemodynamic stability.

Which of the following measures should the nurse focus on for the client with esophageal varices? A. Recognizing hemorrhage B. Controlling blood pressure C. Encouraging nutritional intake D. Teaching the client about varices

A Recognizing the rupture of esophageal varices, or hemorrhage, is the focus of nursing care because the client could succumb to this quickly. Controlling blood pressure is also important because it helps reduce the risk of variceal rupture. It is also important to teach the client what varices are and what foods he should avoid such as spicy foods.

Side effects of loperamide (Imodium) include all of the following except? A. Diarrhea B. Epigastric pain C. Dry mouth D. Anorexia

A Side effects associated with loperamide include CNS fatigue and dizziness, epigastric pain, abdominal cramps, nausea, dry mouth, vomiting, and anorexia. Diarrhea is an indication, not a side effect.

Nurse Juvy is caring for a client with cirrhosis of the liver. To minimize the effects of the disorder, the nurse teaches the client about foods that are high in thiamine. The nurse determines that the client has the best understanding of the dietary measures to follow if the client states an intention to increase the intake of: A. Pork B. Milk C. Chicken D. Broccoli

A The client with cirrhosis needs to consume foods high in thiamine. Thiamine is present in a variety of foods of plant and animal origin. Pork products are especially rich in this vitamin. Other good food sources include nuts, whole grain cereals, and legumes. Milk contains vitamins A, D, and B2. Poultry contains niacin. Broccoli contains vitamins C, E, and K and folic acid

Fistulas are most common with which of the following bowel disorders? A. Crohn's disease B. Diverticulitis C. Diverticulosis D. Ulcerative colitis

A The lesions of Crohn's disease are transmural; that is, they involve all thickness of the bowel. These lesions may perforate the bowel wall, forming fistulas with adjacent structures. Fistulas don't develop in diverticulitis or diverticulosis. The ulcers that occur in the submucosal and mucosal layers of the intestine in ulcerative colitis usually don't progress to fistula formation as in Crohn's disease.

The nurse is doing an admission assessment on a client with a history of duodenal ulcer. To determine whether the problem is currently active, the nurse would assess the client for which of the following most frequent symptom(s) of duodenal ulcer? A. Pain that is relieved by food intake B. Pain that radiated down the right arm C. N/V D. Weight loss

A The most frequent symptom of duodenal ulcer is pain that is relieved by food intake. These clients generally describe the pain as burning, heavy, sharp, or "hungry" pain that often localizes in the midepigastric area. The client with duodenal ulcer usually does not experience weight loss or N/V. These symptoms are usually more typical in the client with a gastric ulcer.

The client with Crohn's disease has a nursing diagnosis of acute pain. The nurse would teach the client to avoid which of the following in managing this problem? A. Lying supine with the legs straight B. Massaging the abdomen C. Using antispasmodic medication D. Using relaxation techniques

A The pain associated with Crohn's disease is alleviated by the use of analgesics and antispasmodics and also is reduced by having the client practice relaxation techniques, applying local cold or heat to the abdomen, massaging the abdomen, and lying with the legs flexed. Lying with the legs extended is not useful because it increases the muscle tension in the abdomen, which could aggravate the inflamed intestinal tissues as the abdominal muscles are stretched.

The nurse is teaching the client how to perform a colostomy irrigation. To enhance the effectiveness of the irrigation and fecal returns, what measure should the nurse instruct the client to do? A. Increase fluid intake B. Reduce the amount of irrigation solution C. Perform the irrigation in the evening D. Place heat on the abdomen

A To enhance effectiveness of the irrigation and fecal returns, the client is instructed to increase fluid intake and prevent constipation.

The nurse is teaching a female client how to perform a colostomy irrigation. To enhance the effectiveness of the irrigation and fecal returns, what measure should the nurse instruct the client to do? A. Increase fluid intake B. Place heat on the abdomen C. Perform the irrigation in the evening D. Reduce the amount of irrigation solution

A To enhance effectiveness of the irrigation and fecal returns, the client is instructed to increase fluid intake and to take other measures to prevent constipation. Options B, C and D will not enhance the effectiveness of this procedure.

When teaching a community group about measures to prevent colon cancer, which instruction should the nurse include? A. "Limit fat intake to 20% to 25% of your total daily calories." B. "Include 15 to 20 grams of fiber into your daily diet." C. "Get an annual rectal examination after age 35." D. "Undergo sigmoidoscopy annually after age 50."

A To help prevent colon cancer, fats should account for no more than 20% to 25% of total daily calories and the diet should include 25 to 30 grams of fiber per day. A digital rectal examination isn't recommended as a stand-alone test for colorectal cancer. For colorectal cancer screening, the American Cancer society advises clients over age 50 to have a flexible sigmoidoscopy every 5 years, yearly fecal occult blood tests, yearly fecal occult blood tests PLUS a flexible sigmoidoscopy every 5 years, a double-contrast barium enema every 5 years, or a colonoscopy every 10 years.

A client has a percutaneous endoscopic gastrostomy tube inserted for tube feedings. Before starting a continuous feeding, the nurse should place the client in which position? A. Semi-Fowlers B. Supine C. Reverse Trendelenburg D. High Fowler's

A To prevent aspiration of stomach contents, the nurse should place the client in semi-Fowler's position. High Fowler's position isn't necessary and may not be tolerated as well as semi-Fowler's.

Nurse Oliver checks for residual before administering a bolus tube feeding to a client with a nasogastric tube and obtains a residual amount of 150 mL. What is appropriate action for the nurse to take? A. Hold the feeding B. Reinstill the amount and continue with administering the feeding C. Elevate the client's head at least 45 degrees and administer the feeding D. Discard the residual amount and proceed with administering the feeding

A Unless specifically indicated, residual amounts more than 100 mL require holding the feeding. Therefore options B, C, and D are incorrect. Additionally, the feeding is not discarded unless its contents are abnormal in color or characteristics

Risk factors for the development of hiatal hernias are those that lead to increased abdominal pressure. Which of the following complications can cause increased abdominal pressure? A. Obesity B. Volvulus C. Constipation D. Intestinal obstruction

A Obesity may cause increased abdominal pressure that pushes the lower portion of the stomach into the thorax.

The nurse would monitor for which of the following adverse reactions to aluminum-containing antacids such as aluminum hydroxide (Amphojel)? A. Diarrhea B. Constipation C. GI upset D. Fluid retention

B Aluminum- and calcium-containing antacids cause constipation, magnesium-containing antacids cause diarrhea, and sodium-containing antacids cause sodium and fluid retention.

The nurse is monitoring a female client receiving paregoric to treat diarrhea for drug interactions. Which drugs can produce additive constipation when given with an opium preparation? a. Antiarrhythmic drugs b. Anticholinergic drugs c. Anticoagulant drugs d. Antihypertensive drugs

Answer B. Paregoric has an additive effect of constipation when used with anticholinergic drugs. Antiarrhythmics, anticoagulants, and antihypertensives aren't known to interact with paregoric

During the first few days of recovery from ostomy surgery for ulcerative colitis, which of the following aspects should be the first priority of client care? A. Body image B. Ostomy care C. Sexual concerns D. Skin care

B Although all of these are concerns the nurse should address, being able to safely manage the ostomy is crucial for the client before discharge.

A male client with extreme weakness, pallor, weak peripheral pulses, and disorientation is admitted to the emergency department. His wife reports that he has been "spitting up blood." A Mallory-Weiss tear is suspected, and the nurse begins taking a client history from the client's wife. The question by the nurse that demonstrates her understanding of Mallory-Weiss tearing is: a. "Tell me about your husband's alcohol usage." b. "Is your husband being treated for tuberculosis?" c. "Has your husband recently fallen or injured his chest?" d. "Describe spices and condiments your husband uses on food."

Answer A. A Mallory-Weiss tear is associated with massive bleeding after a tear occurs in the mucous membrane at the junction of the esophagus and stomach. There is a strong relationship between ethanol usage, resultant vomiting, and a Mallory-Weiss tear. The bleeding is coming from the stomach, not from the lungs as would be true in some cases of tuberculosis. A Mallory-Weiss tear doesn't occur from chest injuries or falls and isn't associated with eating spicy foods.

A male client undergoes total gastrectomy. Several hours after surgery, the nurse notes that the client's nasogastric (NG) tube has stopped draining. How should the nurse respond? a. Notify the physician b. Reposition the tube c. Irrigate the tube d. Increase the suction leveL

Answer A. An NG tube that fails to drain during the postoperative period should be reported to the physician immediately. It may be clogged, which could increase pressure on the suture site because fluid isn't draining adequately. Repositioning or irrigating an NG tube in a client who has undergone gastric surgery can disrupt the anastomosis. Increasing the level of suction may cause trauma to GI mucosa or the suture line.

A male client is recovering from an ileostomy that was performed to treat inflammatory bowel disease. During discharge teaching, the nurse should stress the importance of: a. increasing fluid intake to prevent dehydration. b. wearing an appliance pouch only at bedtime. c. consuming a low-protein, high-fiber diet. d. taking only enteric-coated medications.

Answer A. Because stool forms in the large intestine, an ileostomy typically drains liquid waste. To avoid fluid loss through ileostomy drainage, the nurse should instruct the client to increase fluid intake. The nurse should teach the client to wear a collection appliance at all times because ileostomy drainage is incontinent, to avoid high-fiber foods because they may irritate the intestines, and to avoid enteric-coated medications because the body can't absorb them after an ileostomy

Which diagnostic test would be used first to evaluate a client with upper GI bleeding? a. Endoscopy b. Upper GI series c. Hemoglobin (Hb) levels and hematocrit (HCT) d. Arteriography

Answer A. Endoscopy permits direct evaluation of the upper GI tract and can detect 90% of bleeding lesions. An upper GI series, or barium study, usually isn't the diagnostic method of choice, especially in a client with acute active bleeding who's vomiting and unstable. An upper GI series is also less accurate than endoscopy. Although an upper GI series might confirm the presence of a lesion, it wouldn't necessarily reveal whether the lesion is bleeding. Hb levels and HCT, which indicate loss of blood volume, aren't always reliable indicators of GI bleeding because a decrease in these values may not be seen for several hours. Arteriography is an invasive study associated with life-threatening complications and wouldn't be used for an initial evaluation

A female client who has just been diagnosed with hepatitis A asks, "How could I have gotten this disease?" What is the nurse's best response? a. "You may have eaten contaminated restaurant food." b. "You could have gotten it by using I.V. drugs." c. "You must have received an infected blood transfusion." d. "You probably got it by engaging in unprotected sex.

Answer A. Hepatitis A virus typically is transmitted by the oral-fecal route — commonly by consuming food contaminated by infected food handlers. The virus isn't transmitted by the I.V. route, blood transfusions, or unprotected sex. Hepatitis B can be transmitted by I.V. drug use or blood transfusion. Hepatitis C can be transmitted by unprotected sex.

While a female client is being prepared for discharge, the nasogastric (NG) feeding tube becomes clogged. To remedy this problem and teach the client's family how to deal with it at home, what should the nurse do? a. Irrigate the tube with cola. b. Advance the tube into the intestine. c. Apply intermittent suction to the tube. d. Withdraw the obstruction with a 30-ml syringe.

Answer A. The nurse should irrigate the tube with cola because its effervescence and acidity are suited to the purpose, it's inexpensive, and it's readily available in most homes. Advancing the NG tube is inappropriate because the tube is designed to stay in the stomach and isn't long enough to reach the intestines. Applying intermittent suction or using a syringe for aspiration is unlikely to dislodge the material clogging the tube but may create excess pressure. Intermittent suction may even collapse the tube.

A male client with cholelithiasis has a gallstone lodged in the common bile duct. When assessing this client, the nurse expects to note: a. yellow sclerae. b. light amber urine. c. circumoral pallor. d. black, tarry stools

Answer A. Yellow sclerae may be the first sign of jaundice, which occurs when the common bile duct is obstructed. Urine normally is light amber. Circumoral pallor and black, tarry stools don't occur in common bile duct obstruction; they are signs of hypoxia and GI bleeding, respectively.

When preparing a male client, age 51, for surgery to treat appendicitis, the nurse formulates a nursing diagnosis of Risk for infection related to inflammation, perforation, and surgery. What is the rationale for choosing this nursing diagnosis? a. Obstruction of the appendix may increase venous drainage and cause the appendix to rupture. b. Obstruction of the appendix reduces arterial flow, leading to ischemia, inflammation, and rupture of the appendix. c. The appendix may develop gangrene and rupture, especially in a middle-aged client. d. Infection of the appendix diminishes necrotic arterial blood flow and increases venous drainage.

Answer B. A client with appendicitis is at risk for infection related to inflammation, perforation, and surgery because obstruction of the appendix causes mucus fluid to build up, increasing pressure in the appendix and compressing venous outflow drainage. The pressure continues to rise with venous obstruction; arterial blood flow then decreases, leading to ischemia from lack of perfusion. Inflammation and bacterial growth follow, and swelling continues to raise pressure within the appendix, resulting in gangrene and rupture. Geriatric, not middle-aged, clients are especially susceptible to appendix rupture.

When evaluating a male client for complications of acute pancreatitis, the nurse would observe for: a. increased intracranial pressure. b. decreased urine output. c. bradycardia. d. hypertension

Answer B. Acute pancreatitis can cause decreased urine output, which results from the renal failure that sometimes accompanies this condition. Intracranial pressure neither increases nor decreases in a client with pancreatitis. Tachycardia, not bradycardia, usually is associated with pulmonary or hypovolemic complications of pancreatitis. Hypotension can be caused by a hypovolemic complication, but hypertension usually isn't related to acute pancreatitis.

A female client with dysphagia is being prepared for discharge. Which outcome indicates that the client is ready for discharge? a. The client doesn't exhibit rectal tenesmus. b. The client is free from esophagitis and achalasia. c. The client reports diminished duodenal inflammation. d. The client has normal gastric structures.

Answer B. Dysphagia may be the reason why a client with esophagitis or achalasia seeks treatment. Dysphagia isn't associated with rectal tenesmus, duodenal inflammation, or abnormal gastric structures.

What laboratory finding is the primary diagnostic indicator for pancreatitis? a. Elevated blood urea nitrogen (BUN) b. Elevated serum lipase c. Elevated aspartate aminotransferase (AST) d. Increased lactate dehydrogenase (LD)

Answer B. Elevation of serum lipase is the most reliable indicator of pancreatitis because this enzyme is produced solely by the pancreas. A client's BUN is typically elevated in relation to renal dysfunction; the AST, in relation to liver dysfunction; and LD, in relation to damaged cardiac muscle.

A male client with pancreatitis complains of pain. The nurse expects the physician to prescribe meperidine (Demerol) instead of morphine to relieve pain because: a. meperidine provides a better, more prolonged analgesic effect. b. morphine may cause spasms of Oddi's sphincter. c. meperidine is less addictive than morphine. d. morphine may cause hepatic dysfunction.

Answer B. For a client with pancreatitis, the physician will probably avoid prescribing morphine because this drug may trigger spasms of the sphincter of Oddi (a sphincter at the end of the pancreatic duct), causing irritation of the pancreas. Meperidine has a somewhat shorter duration of action than morphine. The two drugs are equally addictive. Morphine isn't associated with hepatic dysfunction.

A male client with a recent history of rectal bleeding is being prepared for a colonoscopy. How should the nurse position the client for this test initially? a. Lying on the right side with legs straight b. Lying on the left side with knees bent c. Prone with the torso elevated d. Bent over with hands touching the floor

Answer B. For a colonoscopy, the nurse initially should position the client on the left side with knees bent. Placing the client on the right side with legs straight, prone with the torso elevated, or bent over with hands touching the floor wouldn't allow proper visualization of the large intestine

A male client has just been diagnosed with hepatitis A. On assessment, the nurse expects to note: a. severe abdominal pain radiating to the shoulder. b. anorexia, nausea, and vomiting. c. eructation and constipation. d. abdominal ascites.

Answer B. Hallmark signs and symptoms of hepatitis A include anorexia, nausea, vomiting, fatigue, and weakness. Abdominal pain may occur but doesn't radiate to the shoulder. Eructation and constipation are common in gallbladder disease, not hepatitis A. Abdominal ascites is a sign of advanced hepatic disease, not an early sign of hepatitis A.

Which condition is most likely to have a nursing diagnosis of fluid volume deficit? a. Appendicitis b. Pancreatitis c. Cholecystitis d. Gastric ulcer

Answer B. Hypovolemic shock from fluid shifts is a major factor in acute pancreatitis. The other conditions are less likely to exhibit fluid volume deficit.

A male client is recovering from a small-bowel resection. To relieve pain, the physician prescribes meperidine (Demerol), 75 mg I.M. every 4 hours. How soon after administration should meperidine's onset of action occur? a. 5 to 10 minutes b. 15 to 30 minutes c. 30 to 60 minutes d. 2 to 4 hours

Answer B. Meperidine's onset of action is 15 to 30 minutes. It peaks between 30 and 60 minutes and has a duration of action of 2 to 4 hours.

male client has undergone a colon resection. While turning him, wound dehiscence with evisceration occurs. The nurse's first response is to: a. call the physician. b. place saline-soaked sterile dressings on the wound. c. take a blood pressure and pulse. d. pull the dehiscence closed.

Answer B. The nurse should first place saline-soaked sterile dressings on the open wound to prevent tissue drying and possible infection. Then the nurse should call the physician and take the client's vital signs. The dehiscence needs to be surgically closed, so the nurse should never try to close it.

To prevent gastroesophageal reflux in a male client with hiatal hernia, the nurse should provide which discharge instruction? a. "Lie down after meals to promote digestion." b. "Avoid coffee and alcoholic beverages." c. "Take antacids with meals." d. "Limit fluid intake with meals.

Answer B. To prevent reflux of stomach acid into the esophagus, the nurse should advise the client to avoid foods and beverages that increase stomach acid, such as coffee and alcohol. The nurse also should teach the client to avoid lying down after meals, which can aggravate reflux, and to take antacids after eating. The client need not limit fluid intake with meals as long as the fluids aren't gastric irritants.

The nurse is caring for a male client with cirrhosis. Which assessment findings indicate that the client has deficient vitamin K absorption caused by this hepatic disease? a. Dyspnea and fatigue b. Ascites and orthopnea c. Purpura and petechiae d. Gynecomastia and testicular atrophy

Answer C. A hepatic disorder, such as cirrhosis, may disrupt the liver's normal use of vitamin K to produce prothrombin (a clotting factor). Consequently, the nurse should monitor the client for signs of bleeding, including purpura and petechiae. Dyspnea and fatigue suggest anemia. Ascites and orthopnea are unrelated to vitamin K absorption. Gynecomastia and testicular atrophy result from decreased estrogen metabolism by the diseased liver.

Which of the following factors can cause hepatitis A? a. Contact with infected blood b. Blood transfusions with infected blood c. Eating contaminated shellfish d. Sexual contact with an infected person

Answer C. Hepatitis A can be caused by consuming contaminated water, milk, or food — especially shellfish from contaminated water. Hepatitis B is caused by blood and sexual contact with an infected person. Hepatitis C is usually caused by contact with infected blood, including receiving blood transfusions

The nurse is caring for a female client with active upper GI bleeding. What is the appropriate diet for this client during the first 24 hours after admission? a. Regular diet b. Skim milk c. Nothing by mouth d. Clear liquids

Answer C. Shock and bleeding must be controlled before oral intake, so the client should receive nothing by mouth. A regular diet is incorrect. When the bleeding is controlled, the diet is gradually increased, starting with ice chips and then clear liquids. Skim milk shouldn't be given because it increases gastric acid production, which could prolong bleeding. A liquid diet is the first diet offered after bleeding and shock are controlled.

A female client with viral hepatitis A is being treated in an acute care facility. Because the client requires enteric precautions, the nurse should: a. place the client in a private room. b. wear a mask when handling the client's bedpan. c. wash the hands after touching the client. d. wear a gown when providing personal care for the client.

Answer C. To maintain enteric precautions, the nurse must wash the hands after touching the client or potentially contaminated articles and before caring for another client. A private room is warranted only if the client has poor hygiene — for instance, if the client is unlikely to wash the hands after touching infective material or is likely to share contaminated articles with other clients. For enteric precautions, the nurse need not wear a mask and must wear a gown only if soiling from fecal matter is likely.

Nurse Hannah is teaching a group of middle-aged men about peptic ulcers. When discussing risk factors for peptic ulcers, the nurse should mention: a. a sedentary lifestyle and smoking. b. a history of hemorrhoids and smoking. c. alcohol abuse and a history of acute renal failure. d. alcohol abuse and smoking.

Answer D. Risk factors for peptic (gastric and duodenal) ulcers include alcohol abuse, smoking, and stress. A sedentary lifestyle and a history of hemorrhoids aren't risk factors for peptic ulcers. Chronic renal failure, not acute renal failure, is associated with duodenal ulcers.

A female client with hepatitis C develops liver failure and GI hemorrhage. The blood products that would most likely bring about hemostasis in the client are: a. whole blood and albumin. b. platelets and packed red blood cells. c. fresh frozen plasma and whole blood. d. cryoprecipitate and fresh frozen plasma

Answer D. The liver is vital in the synthesis of clotting factors, so when it's diseased or dysfunctional, as in hepatitis C, bleeding occurs. Treatment consists of administering blood products that aid clotting. These include fresh frozen plasma containing fibrinogen and cryoprecipitate, which have most of the clotting factors. Although administering whole blood, albumin, and packed cells will contribute to hemostasis, those products aren't specifically used to treat hemostasis. Platelets are helpful, but the best answer is cryoprecipitate and fresh frozen plasma.

A patient diagnosed with ulcerative colitis is prescribed the aminosalicylate sulfasalazine. When teaching the patient about this medication, which of the following statements is a priority for the healthcare provider include? Please choose from one of the following options. A. "Be sure to limit your intake of fluids during therapy." B. "Avoid exposure to sunlight while taking this medication." C. "Call our office immediately if your urine turns an orangish color." D. "You may crush the enteric-coated tablet and mix it with applesauce."

B

Digoxin preparations and absorbents should not be given simultaneously. As a nurse, you are aware that if these agents are given simultaneously, which of the following will occur? A. Increased absorption of digoxin B. Decreased absorption of digoxin C. Increased absorption of the absorbent D. Decreased absorption of the absorbent

B

In teaching patients at risk for upper GI bleeding to prevent bleeding episodes, the nurse stresses that: a. all stools and vomit must be tested for blood b. the use of over the counter meds of any kind should be avoided c. antacids should be taken with all prescribed meds d. Cytotec should be used to protect gastric mucosa

B

Patient with an ulcer of the posterior portion of duodenum experiences: a. pain that occurs after not eating all day b. back pain that occurs 2-4 hrs after eating c. midepigastric pain unrelieved with antacids d. high epigastric burning relieved with food intake

B

Upon examining a patient 8 hrs after formation of a colostomy the nurse would expect to find a. hypoactive, high pitched bowel sounds b. brickred, puffy stoma that oozes blood c. purplish stoma, shiny and moist d. small amt of liquid fecal drainage from stomA

B

Which of the following tests should be administered to a client suspected of having diverticulosis? A. Abdominal ultrasound B. Barium enema C. Barium swallow D. Gastroscopy

B A barium enema will cause diverticula to fill with barium and be easily seen on x-ray. An abdominal US can tell more about structures, such as the gallbladder, liver, and spleen, than the intestine. A barium swallow and gastroscopy view upper GI structures.

Which of the following nursing diagnoses is appropriate for a patient receiving famotidine (Pepcid)? A. Increased risk for infection due to immunosuppression B. Potential risk for bleeding related to thrombocytopenia C. Alteration in urinary elimination related to retention D. Alteration in tissue perfusion related to hypertension

B A serious side effect of famotidine is thrombocytopenia, which is manifested by a decrease in platelet count and an increased risk of bleeding.

The nurse instructs the nursing assistant on how to provide oral hygiene for a client who cannot perform this task for himself. Which of the following techniques should the nurse tell the assistant to incorporate into the client's daily care? A. Assess the oral cavity each time mouth care is given and record observations B. Use a soft toothbrush to brush the client's teeth after each meal C. Swab the client's tongue, gums, and lips with a soft foam applicator every 2 hours D. Rinse the client's mouth with mouthwash several times a day

B A soft toothbrush should be used to brush the client's teeth after each meal and more often as needed. Mechanical cleaning is necessary to maintain oral health, simulate gingiva, and remove plaque. Assessing the oral cavity and recording observations is the responsibility of the nurse, not the nursing assistant. Swabbing with a safe foam applicator does not provide enough friction to clean the mouth. Mouthwash can be a drying irritant and is not recommended for frequent use.

Which of the following symptoms would a client in the early stages of peritonitis exhibit? A. Abdominal distention B. Abdominal pain and rigidity C. Hyperactive bowel sounds D. Right upper quadrant pain

B Abdominal pain causing rigidity of the abdominal muscles is characteristic of peritonitis. Abdominal distention may occur as a late sign but not early on. Bowel sounds may be normal or decreased but not increased. Right upper quadrant pain is characteristic of cholecystitis or hepatitis.

Which of the following tests is most commonly used to diagnose cholecystitis? A. Abdominal CT scan B. Abdominal ultrasound C. Barium swallow D. Endoscopy

B An abdominal ultrasound can show if the gallbladder is enlarged, if gallstones are present, if the gallbladder wall is thickened, or if distention of the gallbladder lumen is present. An abdominal CT scan can be used to diagnose cholecystitis, but it usually isn't necessary. A barium swallow looks at the stomach and the duodenum. Endoscopy looks at the esophagus, stomach, and duodenum.

If a gastric acid perforates, which of the following actions should not be included in the immediate management of the client? A. Blood replacement B. Antacid administration C. Nasogastric tube suction D. Fluid and electrolyte replacement

B Antacids aren't helpful in perforation. The client should be treated with antibiotics as well as fluid, electrolyte, and blood replacement. NG tube suction should also be performed to prevent further spillage of stomach contents into the peritoneal cavity.

For Jayvin who is taking antacids, which instruction would be included in the teaching plan? A. "Take the antacids with 8 oz of water." B. "Avoid taking other medications within 2 hours of this one." C. "Continue taking antacids even when pain subsides." D. "Weigh yourself daily when taking this medication.

B Antacids neutralize gastric acid and decrease the absorption of other medications. The client should be instructed to avoid taking other medications within 2 hours of the antacid. Water, which dilutes the antacid, should not be taken with antacid. A histamine receptor antagonist should be taken even when pain subsides. Daily weights are indicated if the client is taking a diuretic, not an antacid.

A client with a peptic ulcer reports epigastric pain that frequently awakens her at night, a feeling of fullness in the abdomen, and a feeling of anxiety about her health. Based on this information, which nursing diagnosis would be most appropriate? A. Imbalanced Nutrition: Less than Body Requirements related to anorexia B. Disturbed Sleep Pattern related to epigastric pain C. Ineffective Coping related to exacerbation of duodenal ulcer D. Activity Intolerance related to abdominal pain

B Based on the data provided, the most appropriate nursing diagnosis would be Disturbed Sleep pattern. A client with a duodenal ulcer commonly awakens at night with pain. The client's feelings of anxiety do not necessarily indicate that she is coping ineffectively.

A nurse is monitoring a client admitted to the hospital with a diagnosis of appendicitis. The client is scheduled for surgery in 2 hours. The client begins to complain of increased abdominal pain and begins to vomit. On assessment the nurse notes that the abdomen is distended and the bowel sounds are diminished. Which of the following is the most appropriate nursing intervention? A. Administer dilaudid B. Notify the physician C. Call and ask the operating room team to perform the surgery as soon as possible D. Reposition the client and apply a heating pad on a warm setting to the client's abdomen

B Based on the signs and symptoms presented in the question, the nurse should suspect peritonitis and should notify the physician. Administering pain medication is not an appropriate intervention. Heat should never be applied to the abdomen of a client with suspected appendicitis. Scheduling surgical time is not within the scope of nursing practice, although the physician probably would perform the surgery earlier than the prescheduled time.

A client with peptic ulcer disease tells the nurse that he has black stools, which he has not reported to his physician. Based on this information, which nursing diagnosis would be appropriate for this client? A. Ineffective coping related to fear of diagnosis of chronic illness B. Deficient knowledge related to unfamiliarity with significant signs and symptoms C. Constipation related to decreased gastric motility D. Imbalanced nutrition: Less than body requirements due to gastric bleeding

B Black, tarry stools are an important warning sign of bleeding in peptic ulcer disease. Digested blood in the stomach causes it to be black. The odor of the stool is very stinky. Clients with peptic ulcer disease should be instructed to report the incidence of black stools promptly to their physician.

he nurse is caring for a male client with a diagnosis of chronic gastritis. The nurse monitors the client knowing that this client is at risk for which vitamin deficiency? A. Vitamin A B. Vitamin B12 C. Vitamin C D. Vitamin E

B Chronic gastritis causes deterioration and atrophy of the lining of the stomach, leading to the loss of the function of the parietal cells. The source of the intrinsic factor is lost, which results in the inability to absorb vitamin B12. This leads to the development of pernicious anemia. The client is not at risk for vitamin A, C, or E deficiency.

For Rico who has chronic pancreatitis, which nursing intervention would be most helpful? A. Allowing liberalized fluid intake B. Counseling to stop alcohol consumption C. Encouraging daily exercise D. Modifying dietary protein

B Chronic pancreatitis typically results from repeated episodes of acute pancreatitis. More than half of chronic pancreatitis cases are associated with alcoholism. Counseling to stop alcohol consumption would be the most helpful for the client. Dietary protein modification is not necessary for chronic pancreatitis. Daily exercise and liberalizing fluid intake would be helpful but not the most beneficial intervention.

The nurse is reviewing the record of a client with Crohn's disease. Which of the following stool characteristics would the nurse expect to note documented on the client's record? A. Chronic constipation B. Diarrhea C. Constipation alternating with diarrhea D. Stool constantly oozing from the rectum

B Crohn's disease is characterized by nonbloody diarrhea of usually not more than four to five stools daily. Over time, the diarrhea episodes increase in frequency, duration and severity. The other option are not associated with diarrhea.

Which goal of the client's care should take priority during the first days of hospitalization for an exacerbation of ulcerative colitis? A. Promoting self-care and independence B. Managing diarrhea C. Maintaining adequate nutrition D. Promoting rest and comfort

B Diarrhea is the primary symptom in an exacerbation of ulcerative colitis, and decreasing the frequency of stools is the first goal of treatment. The other goals are ongoing and will be best achieved by halting the exacerbation. The client may receive antidiarrheal medications, antispasmodic agents, bulk hydrophilic agents, or anti-inflammatory drugs.

Which of the following definitions best describes diverticulosis? A. An inflamed outpouching of the intestine B. A noninflamed outpouching of the intestine C. The partial impairment of the forward flow of intestinal contents D. An abnormal protrusion of an organ through the structure that usually holds it.

B Diverticulosis involves a noninflamed outpouching of the intestine. Diverticulitis involves an inflamed outpouching. The partial impairment of forward flow of the intestine is an obstruction; abnormal protrusion of an organ is a hernia.

The nurse would assess the client experiencing an acute episode of cholecysitis for pain that is located in the right A. Upper quadrant and radiates to the left scapula and shoulder B. Upper quadrant and radiates to the right scapula and shoulder C. Lower quadrant and radiates to the umbilicus D. Lower quadrant and radiates to the back

B During an acute "gallbladder attack," the client may complain of severe right upper quadrant pain that radiates to the right scapula and shoulder. This is governed by the pattern on dermatomes in the body.

An enema is prescribed for a client with suspected appendicitis. Which of the following actions should the nurse take? A. Prepare 750 ml of irrigating solution warmed to 100*F B. Question the physician about the order C. Provide privacy and explain the procedure to the client D. Assist the client to left lateral Sim's position

B Enemas are contraindicated in an acute abdominal condition of unknown origin as well as after recent colon or rectal surgery or myocardial infarction. The other answers are correct only when enema administration is appropriate.

The nurse is assessing a client 24 hours following a cholecystectomy. The nurse notes that the T-tube has drained 750ml of green-brown drainage. Which nursing intervention is most appropriate? A. Notify the physician B. Document the findings C. Irrigate the T-tube D. Clamp the T-tube

B Following cholecystectomy, drainage from the T-tube is initially bloody and then turns to green-brown. The drainage is measured as output. The amount of expected drainage will range from 500 to 1000 ml per day. The nurse would document the output.

When planning home care for a client with hepatitis A, which preventive measure should be emphasized to protect the client's family? A. Keeping the client in complete isolation B. Using good sanitation with dishes and shared bathrooms C. Avoiding contact with blood-soiled clothing or dressing D. Forbidding the sharing of needles or syringes

B Hepatitis A is transmitted through the fecal oral route or from contaminated water or food. Measures to protect the family include good handwashing, personal hygiene and sanitation, and use of standard precautions. Complete isolation is not required. Avoiding contact with blood-soiled clothing or dressings or avoiding the sharing of needles or syringes are precautions needed to prevent transmission of hepatitis B.

The nurse is performing a colostomy irrigation on a male client. During the irrigation, the client begins to complain of abdominal cramps. What is the appropriate nursing action? A. Notify the physician B. Stop the irrigation temporarily C. Increase the height of the irrigation D. Medicate for pain and resume the irrigation

B If cramping occurs during a colostomy irrigation, the irrigation flow is stopped temporarily and the client is allowed to rest. Cramping may occur from an infusion that is too rapid or is causing too much pressure. The physician does not need to be notified. Increasing the height of the irrigation will cause further discomfort. Medicating the client for pain is not the appropriate action in this situation.

Which of the following symptoms is associated with ulcerative colitis? A. Dumping syndrome B. Rectal bleeding C. Soft stools D. Fistulas

B In ulcerative colitis, rectal bleeding is the predominant symptom. Soft stools are more commonly associated with Crohn's disease, in which malabsorption is more of a problem. Dumping syndrome occurs after gastric surgeries. Fistulas are associated with Crohn's disease.

The nurse is reviewing the medication record of a client with acute gastritis. Which medication, if noted on the client's record, would the nurse question? A. Digoxin (Lanoxin) B. Indomethacin (Indocin) C. Furosemide (Lasix) D. Propranolol hydrochloride (Inderal)

B Indomethacin (Indocin) is a NSAID and can cause ulceration of the esophagus, stomach, duodenum, or small intestine. Indomethacin is contraindicated in a client with GI disorders.

A client is suspected of having hepatitis. Which diagnostic test result will assist in confirming this diagnosis? A. Elevated hemoglobin level B. Elevated serum bilirubin level C. Elevated blood urea nitrogen level D. Decreased erythrocyte sedimentation rate

B Laboratory indicators of hepatitis include elevated liver enzyme levels, elevated serum bilirubin levels, elevated erythrocyte sedimentation rates, and leukopenia. An elevated blood urea nitrogen level may indicate renal dysfunction. A hemoglobin level is unrelated to this diagnosis.

When teaching a client about pancreatic function, the nurse understands that pancreatic lipase performs which function? A. Transports fatty acids into the brush border B. Breaks down fat into fatty acids and glycerol C. Triggers cholecystokinin to contract the gallbladder D. Breaks down protein into dipeptides and amino acids

B Lipase hydrolyses or breaks down fat into fatty acids and glycerol. Lipase is not involved with the transport of fatty acids into the brush border. Fat itself triggers cholecystokinin release. Protein breakdown into dipeptides and amino acids is the function of trypsin, not lipase.

A female client complains of gnawing epigastric pain for a few hours after meals. At times, when the pain is severe, vomiting occurs. Specific tests are indicated to rule out: A. Cancer of the stomach B. Peptic ulcer disease C. Chronic gastritis D. Pylorospasm

B Peptic ulcer disease is characteristically gnawing epigastric pain that may radiate to the back. Vomiting usually reflects pyloric spasm from muscular spasm or obstruction. Cancer (1) would not evidence pain or vomiting unless the pylorus was obstructed.

Which of the following best describes the method of action of medications, such as ranitidine (Zantac), which are used in the treatment of peptic ulcer disease? A. Neutralize acid B. Reduce acid secretions C. Stimulate gastrin release D. Protect the mucosal barrier

B Ranitidine is a histamine-2 receptor antagonist that reduces acid secretion by inhibiting gastrin secretion.

Which of the following factors is believed to cause ulcerative colitis? A. Acidic diet B. Altered immunity C. Chronic constipation D. Emotional stress

B Several theories exist regarding the cause of ulcerative colitis. One suggests altered immunity as the cause based on the extraintestinal characteristics of the disease, such as peripheral arthritis and cholangitis. Diet and constipation have no effect on the development of ulcerative colitis. Emotional stress can exacerbate the attacks but isn't believed to be the primary cause.

After a right hemicolectomy for treatment of colon cancer, a 57-year old client is reluctant to turn while on bed rest. Which action by the nurse would be appropriate? A. Asking a coworker to help turn the client B. Explaining to the client why turning is important C. Allowing the client to turn when he's ready to do so D. Telling the client that the physician's order states he must turn every 2 hours

B The appropriate action is to explain the importance of turning to avoid postoperative complications. Asking a coworker to help turn the client would infringe on his rights. Allowing him to turn when he's ready would increase his risk for postoperative complications. Telling him he must turn because of the physician's orders would put him on the defensive and exclude him from participating in care decision.

Which of the following factors should be the main focus of nursing management for a client hospitalized for cholecystitis? A. Administration of antibiotics B. Assessment for complications C. Preparation for lithotripsy D. Preparation for surgery

B The client with acute cholecystitis should first be monitored for perforation, fever, abscess, fistula, and sepsis. After assessment, antibiotics will be administered to reduce the infection. Lithotripsy is used only for a small percentage of clients. Surgery is usually done after the acute infection has subsided.

For a client with hepatic cirrhosis who has altered clotting mechanisms, which intervention would be most important? A. Allowing complete independence of mobility B. Applying pressure to injection sites C. Administering antibiotics as prescribed D. Increasing nutritional intake

B The client with cirrhosis who has altered clotting is at high risk for hemorrhage. Prolonged application of pressure to injection or bleeding sites is important. Complete independence may increase the client's potential for injury, because an unsupervised client may injure himself and bleed excessively. Antibiotics and good nutrition are important to promote liver regeneration. However, they are not most important for a client at high risk for hemorrhage.

A client with irritable bowel syndrome is being prepared for discharge. Which of the following meal plans should the nurse give the client? A. Low fiber, low-fat B. High fiber, low-fat C. Low fiber, high-fat D. High-fiber, high-fat

B The client with irritable bowel syndrome needs to be on a diet that contains at least 25 grams of fiber per day. Fatty foods are to be avoided because they may precipitate symptoms.

Which of the following symptoms is a client with colon cancer most likely to exhibit? A. A change in appetite B. A change in bowel habit C. An increase in body weight D. An increase in body temperature

B The most common complaint of the client with colon cancer is a change in bowel habits. The client may have anorexia, secondary abdominal distention, or weight loss. Fever isn't associated with colon cancer.

The nurse is providing discharge instructions to a client following gastrectomy. Which measure will the nurse instruct the client to follow to assist in preventing dumping syndrome? A. Eat high-carbohydrate foods B. Limit the fluids taken with meals C. Ambulate following a meal D. Sit in a high-Fowlers position during meals

B The nurse should instruct the client to decrease the amount of fluid taken at meals and to avoid high carbohydrate foods including fluids such as fruit nectars; to assume a low-Fowler's position during meals; to lie down for 30 minutes after eating to delay gastric emptying; and to take antispasmodics as prescribed.

Spironolactone (Aldactone) is prescribed for a client with chronic cirrhosis and ascites. The nurse should monitor the client for which of the following medication-related side effects? A. Jaundice B. Hyperkalemia C. Tachycardia D. Constipation

B This is a potassium-sparing diuretic so clients should be monitored closely for hyperkalemia. Diarrhea, dizziness, and headaches are other more common side effects. Tachycardia, jaundice, and constipation are not expected side effects of spironolactone (Aldactone).

Which of the following will the nurse include in the care plan for a client hospitalized with viral hepatitis? A. Increase fluid intake to 3000 ml per day B. Adequate bed rest C. Bland diet D. Administer antibiotics as ordered

B Treatment of hepatitis consists of bed rest during the acute phase to reduce metabolic demands on the liver, thus increasing blood supply and cell regeneration. Forcing fluids, antibiotics, and bland diets are not part of the treatment plan for viral hepatitis.

The nurse evaluates the client's stoma during the initial post-op period. Which of the following observations should be reported immediately to the physician? A. The stoma is slightly edematous B. The stoma is dark red to purple C. The stoma oozes a small amount of blood D. The stoma does not expel stool

B A dark red to purple stoma indicates inadequate blood supply. Mild edema and slight oozing of blood are normal in the early post-op period. The colostomy would typically not begin functioning until 2-4 days after surgery.

While caring for a client with peptic ulcer disease, the client reports that he has been nauseated most of the day and is now feeling lightheaded and dizzy. Based upon these findings, which nursing actions would be most appropriate for the nurse to take? Select all that apply. A. Administering an antacid hourly until nausea subsides. B. Monitoring the client's vital signs C. Notifying the physician of the client's symptoms D. Initiating oxygen therapy E. Reassessing the client on an hour

B,C The symptoms of nausea and dizziness in a client with peptic ulcer disease may be indicative of hemorrhage and should not be ignored. The appropriate nursing actions at this time are for the nurse to monitor the client's vital signs and notify the physician of the client's symptoms. To administer an antacid hourly or to wait one hour to reassess the client would be inappropriate; prompt intervention is essential in a client who is potentially experiencing a gastrointestinal hemorrhage. The nurse would notify the physician of assessment findings and then initiate oxygen therapy if ordered by the physician.

Proton pump inhibitors: A. Gastric ulcer formation B. GERD C. Achlorhydria D. Diverticulosis

C Because the proton pump inhibitors stop the final step of acid secretion, they can block up to 90% of acid secretion, leading to achlorhydria (without acid).

A client diagnosed with chronic cirrhosis who has ascites and pitting peripheral edema also has hepatic encephalopathy. Which of the following nursing interventions are appropriate to prevent skin breakdown? (Select all that apply.) A. Range of motion every 4 hours B. Turn and reposition every 2 hours C. Abdominal and foot massages every 2 hours D. Alternating air pressure mattress E. Sit in chair for 30 minutes each shift

B,D Edematous tissue must receive meticulous care to prevent tissue breakdown. Range of motion exercises preserve joint function but do not prevent skin breakdown. Abdominal or foot massage will not prevent skin breakdown but must be cleansed carefully to prevent breaks in skin integrity. The feet should be kept at the level of heart or higher so Fowler's position should be employed. An air pressure mattress, careful repositioning can prevent skin breakdown.

The nurse is caring for a client who has had a gastroscopy. Which of the following symptoms may indicate that the client is developing a complication related to the procedure? Select all that apply. A. The client complains of a sore throat B. The client has a temperature of 100*F C. The client appears drowsy following the procedure D. The client complains of epigastric pain E. The client experiences hematemesis

B,D,E Following a gastroscopy, the nurse should monitor the client for complications, which include perforation and the potential for aspiration. An elevated temperature, complaints of epigastric pain, or the vomiting of blood (hematemesis) are all indications of a possible perforation and should be reported promptly. A sore throat is a common occurrence following a gastroscopy. Clients are usually sedated to decrease anxiety and the nurse would anticipate that the client will be drowsy following the procedure.

When planning care for a client with ulcerative colitis who is experiencing symptoms, which client care activities can the nurse appropriately delegate to a unlicensed assistant? Select all that apply. A. Assessing the client's bowel sounds B. Providing skin care following bowel movements C. Evaluating the client's response to antidiarrheal medications D. Maintaining intake and output records E. Obtaining the client's weigh

B,D,E The nurse can delegate the following basic care activities to the unlicensed assistant: providing skin care following bowel movements, maintaining intake and output records, and obtaining the client's weight. Assessing the client's bowel sounds and evaluating the client's response to medication are registered nurse activities that cannot be delegated.

A patient with advanced cirrhosis has a nursing diagnosis of imbalanced nutrition: less than body requirements r/t anorexia and inadequate food intake. An appropriate midday snack for the patient would be a. peanut butter and salt free crackers b. a fresh tomato sandwich with salt free butter c. popcorn with salt free butter and herbal seasoning d. canned chicken noodle soup with low protein bread

B- The patient with advanced, complicated cirrhosis requires a high calore, high carbohydrate diet with moderate to low fat. Patients with cirrhosis are at risk for edema and ascites and their sodium intake should be limited. The tomato sandwich with salt free butter best meets these requirements. Rough foods, such as popcorn, may irritate the esophagus and stomach and lead to bleeding. Peanut butter is high in sodium and fat, and canned chicken noodle soup is very high in sodium

Patient with pancreatitis has nursing dx of pain r/t distention of pancreas and peritoneal irritation. In addition to effective use of analgesics the nurse should: a. provide diversional activiies to distract patient b. provide small frequent meals c. position the patient on the side with the head of the bed elevated 45º d. ambulate the patient q 3-4 hours

C

The healthcare provider is assessing a patient diagnosed with ulcerative colitis. The patient has an altered level of consciousness, fever, and lower abdominal distension. Which of these additional findings would confirm a diagnosis of toxic megacolon? Please choose from one of the following options. A. Bradycardia B. Splenomegaly C. Leukocytosis D. Constipation

C

When assessing a patient with pancreatitits, nurse would expect to find: a. hyperactive bowel sounds b. hypertension and tachycardia c. severe midepigastric of LUQ pain d. temp greater than 102º

C

he nurse determines tha further discharge instruction is needed whne the patient with acute pancreatititis states: a. "i should observe for fat in my stools" b. "I must not use alcohol to prevent future attacks" c. "I shouldn't eat salty foods" d. "I will need to continue to monitor my blood glucose levels until my pancreas is healed"

C

The hospitalized client with GERD is complaining of chest discomfort that feels like heartburn following a meal. After administering an ordered antacid, the nurse encourages the client to lie in which of the following positions? A. Supine with the head of the bed flat B. On the stomach with the head flat C. On the left side with the head of the bed elevated 30 degrees D. On the right side with the head of the bed elevated 30 degrees

C The discomfort of reflux is aggravated by positions that compress the abdomen and the stomach. These include lying flat on the back or on the stomach after a meal of lying on the right side. The left side-lying position with the head of the bed elevated is most likely to give relief to the client.

Which of the following tests can be performed to diagnose a hiatal hernia? A. Colonoscopy B. Lower GI series C. Barium swallow D. Abdominal x-rays

C A barium swallow with fluoroscopy shows the position of the stomach in relation to the diaphragm. A colonoscopy and a lower GI series show disorders of the intestine.

In a client with diarrhea, which outcome indicates that fluid resuscitation is successful? A. The client passes formed stools at regular intervals B. The client reports a decrease in stool frequency and liquidity C. The client exhibits firm skin turgor D. The client no longer experiences perianal burning

C A client with diarrhea has a nursing diagnosis of Deficient fluid volume related to excessive fluid loss in the stool. Expected outcomes include firm skin turgor, moist mucous membranes, and urine output of at least 30 ml/hr. The client also has a nursing diagnosis of diarrhea, with expected outcomes of passage of formed stools at regular intervals and a decrease in stool frequency and liquidity. The client is at risk for impaired skin integrity related to irritation from diarrhea; expected outcomes for this diagnosis include absence of erythema in perianal skin and mucous membranes and absence of perianal tenderness or burning.

In a client with Crohn's disease, which of the following symptoms should not be a direct result from antibiotic therapy? A. Decrease in bleeding B. Decrease in temperature C. Decrease in body weight D. Decrease in the number of stools

C A decrease in body weight may occur during therapy due to inadequate dietary intake, but isn't related to antibiotic therapy. Effective antibiotic therapy will be noted by a decrease in temperature, number of stools, and bleeding.

The client has just had surgery to create an ileostomy. The nurse assesses the client in the immediate post-op period for which of the following most frequent complications of this type of surgery? A. Intestinal obstruction B. Fluid and electrolyte imbalance C. Malabsorption of fat D. Folate deficiency

C A major complication that occurs most frequent following an ileostomy is fluid and electrolyte imbalance. The client requires constant monitoring of intake and output to prevent this from happening. Losses require replacement by intravenous infusion until the client can tolerate a diet orally. Intestinal obstruction is a less frequent complication. Fat malabsorption and folate deficiency are complications that could occur later in the postoperative period.

A male client with a peptic ulcer is scheduled for a vagotomy and the client asks the nurse about the purpose of this procedure. Which response by the nurse best describes the purpose of a vagotomy? A. Halts stress reactions B. Heals the gastric mucosa C. Reduces the stimulus to acid secretions D. Decreases food absorption in the stomach

C A vagotomy, or cutting of the vagus nerve, is done to eliminate parasympathetic stimulation of gastric secretion. Options A, B, and D are incorrect descriptions of a vagotomy.

Which of the following factors is believed to be linked to Crohn's disease? A. Constipation B. Diet C. Hereditary D. Lack of exercise

C Although the definite cause of Crohn's disease is unknown, it's thought to be associated with infectious, immune, or psychological factors. Because it has a higher incidence in siblings, it may have a genetic cause.

When a client has peptic ulcer disease, the nurse would expect a priority intervention to be: A. Assisting in inserting a Miller-Abbott tube B. Assisting in inserting an arterial pressure line C. Inserting a nasogastric tube D. Inserting an I.V

C An NG tube insertion is the most appropriate intervention because it will determine the presence of active GI bleeding. A Miller-Abbott tube (1) is a weighted, mercury-filled ballooned tube used to resolve bowel obstructions. There is no evidence of shock or fluid overload in the client; therefore, an arterial line (2) is not appropriate at this time and an IV (4) is optional.

Medical management of the client with diverticulitis should include which of the following treatments? A. Reduced fluid intake B. Increased fiber in diet C. Administration of antibiotics D. Exercises to increase intra-abdominal pressure

C Antibiotics are used to reduce the inflammation. The client isn't typically isn't allowed anything orally until the acute episode subsides. Parenteral fluids are given until the client feels better; then it's recommended that the client drink eight 8-ounce glasses of water per day and gradually increase fiber in the diet to improve intestinal motility. During the acute phase, activities that increase intra-abdominal pressure should be avoided to decrease pain and the chance of intestinal obstruction.

The most important pathophysiologic factor contributing to the formation of esophageal varices is: A. Decreased prothrombin formation B. Decreased albumin formation by the liver C. Portal hypertension D. Increased central venous pressure

C As the liver cells become fatty and degenerate, they are no longer able to accommodate the large amount of blood necessary for homeostasis. The pressure in the liver increases and causes increased pressure in the venous system. As the portal pressure increases, fluid exudes into the abdominal cavity. This is called ascites.

Which of the following associated disorders may the client with Crohn's disease exhibit? A. Ankylosing spondylitis B. Colon cancer C. Malabsorption D. Lactase deficiency

C Because of the transmural nature of Crohn's disease lesions, malaborption may occur with Crohn's disease. Ankylosing spondylitis and colon cancer are more commonly associated with ulcerative colitis. Lactase deficiency is caused by a congenital defect in which an enzyme isn't present.

The nurse is preparing a discharge teaching plan for the client who had an umbilical hernia repair. Which of the following would the nurse include in the plan? A. Restricting pain medication B. Maintaining bedrest C. Avoiding coughing D. Irrigating the drain

C Bedrest is not required following this surgical procedure. The client should take analgesics as needed and as prescribed to control pain. A drain is not used in this surgical procedure, although the client may be instructed in simple dressing changes. Coughing is avoided to prevent disruption of the tissue integrity, which can occur because of the location of this surgical procedure.

The mechanism of action of diphenoxylate (Lotomil) is: A. An increase in intestinal excretion of water B. An increase in intestinal motility C. A decrease in peristalsis in the intestinal wall D. A decrease in the reabsorption of water in the boweL

C Diphenoxylate acts on the smooth muscle of the intestinal tract to inhibit GI motility and excessive propulsion of the GI tract (peristalsis).

he nurse is providing discharge instructions to a male client following gastrectomy and instructs the client to take which measure to assist in preventing dumping syndrome? A. Ambulate following a meal B. Eat high carbohydrate foods C. Limit the fluid taken with meal D. Sit in a high-Fowler's position during meals

C Dumping syndrome is a term that refers to a constellation of vasomotor symptoms that occurs after eating, especially following a Billroth II procedure. Early manifestations usually occur within 30 minutes of eating and include vertigo, tachycardia, syncope, sweating, pallor, palpitations, and the desire to lie down. The nurse should instruct the client to decrease the amount of fluid taken at meals and to avoid high-carbohydrate foods, including fluids such as fruit nectars; to assume a low-Fowler's position during meals; to lie down for 30 minutes after eating to delay gastric emptying; and to take antispasmodics as prescribed.

The nurse is monitoring a client for the early signs of dumping syndrome. Which symptom indicates this occurrence? A. Abdominal cramping and pain B. Bradycardia and indigestion C. Sweating and pallor D. Double vision and chest pain

C Early manifestations of dumping syndrome occur 5 to 30 minutes after eating. Symptoms include vertigo, tachycardia, syncope, sweating, pallor, palpitations, and the desire to lie down.

Which of the following symptoms is common with a hiatal hernia? A. Left arm pain B. Lower back pain C. Esophageal reflux D. Abdominal cramping

C Esophageal reflux is a common symptom of hiatal hernia. This seems to be associated with chronic exposure of the lower esophageal sphincter to the lower pressure of the thorax, making it less effective.

A client's ulcerative colitis symptoms have been present for longer than 1 week. The nurse recognizes that the client should be assessed carefully for signs of which of the following complications? A. Heart failure B. DVT C. Hypokalemia D. Hypocalcemia

C Excessive diarrhea causes significant depletion of the body's stores of sodium and potassium as well as fluid. The client should be closely monitored for hypokalemia and hyponatremia. Ulcerative colitis does not place the client at risk for heart failure, DVT, or hypocalcemia.

Five days after undergoing surgery, a client develops a small-bowel obstruction. A Miller-Abbott tube is inserted for bowel decompression. Which nursing diagnosis takes priority? A. Imbalanced nutrition: Less than body requirements B. Acute pain C. Deficient fluid volume D. Excess fluid volume

C Fluid shifts to the site of the bowel obstruction, causing a fluid deficit in the intravascular spaces. If the obstruction isn't resolved immediately, the client may experience an imbalanced nutritional status (less than body requirements); however, deficient fluid volume takes priority. The client may also experience pain, but that nursing diagnosis is also of lower priority than deficient fluid volume.

A client who has ulcerative colitis has persistent diarrhea. He is thin and has lost 12 pounds since the exacerbation of his ulcerative colitis. The nurse should anticipate that the physician will order which of the following treatment approaches to help the client meet his nutritional needs? A. Initiate continuous enteral feedings B. Encourage a high protein, high-calorie diet C. Implement total parenteral nutrition D. Provide six small meals a day

C Food will be withheld from the client with severe symptoms of ulcerative colitis to rest the bowel. To maintain the client's nutritional status, the client will be started on TPN. Enteral feedings or dividing the diet into 6 small meals does not allow the bowel to rest. A high-calorie, high-protein diet will worsen the client's symptoms.

The nurse has given instructions to the client with an ileostomy about foods to eat to thicken the stool. The nurse determines that the client needs further instructions if the client stated to eat which of the following foods to make the stools less watery? A. Pasta B. Boiled rice C. Bran D. Low-fat cheese

C Foods that help thicken the stool of the client with an ileostomy include pasta, boiled rice, and low-fat cheese. Bran is high in dietary fiber and thus will increase output of watery stool by increasing propulsion through the bowel. Ileostomy output is liquid. Addition or elimination of various foods can help thicken or loosen this liquid drainage.

Marie, a 51-year-old woman, is diagnosed with cholecystitis. Which diet, when selected by the client, indicates that the nurse's teaching has been successful? A. 4-6 small meals of low-carbohydrate foods daily B. High-fat, high-carbohydrate meals C. Low-fat, high-carbohydrate meals D. High-fat, low protein meals

C For the client with cholecystitis, fat intake should be reduced. The calories from fat should be substituted with carbohydrates. Reducing carbohydrate intake would be contraindicated. Any diet high in fat may lead to another attack of cholecystitis.

A male client who is recovering from surgery has been advanced from a clear liquid diet to a full liquid diet. The client is looking forward to the diet change because he has been "bored" with the clear liquid diet. The nurse would offer which full liquid item to the client? A. Tea B. Gelatin C. Custard D. Popsicle

C Full liquid food items include items such as plain ice cream, sherbet, breakfast drinks, milk, pudding and custard, soups that are strained, and strained vegetable juices. A clear liquid diet consists of foods that are relatively transparent. The food items in options A, B, and D are clear liquids.

A patient unable to tolerate oral medications may be prescribed which of the following proton pump inhibitors to be administered intravenously? A. lansoprazole (Prevacid) B. omeprazole (Prilosec) C. pantoprazole (Protonix) D. esomeprazole (Nexium)

C Pantoprazole is the only proton pump inhibitor that is available for intravenous administration. The other medications in this category may only be administered orally.

Which of the following definitions best describes gastritis? A. Erosion of the gastric mucosa B. Inflammation of a diverticulum C. Inflammation of the gastric mucosa D. Reflux of stomach acid into the esophagus

C Gastritis is an inflammation of the gastric mucosa that may be acute (often resulting from exposure to local irritants) or chronic (associated with autoimmune infections or atrophic disorders of the stomach). Erosion of the mucosa results in ulceration. Inflammation of a diverticulum is called diverticulitis; reflux of stomach acid is known as gastroesophageal disease.

A client with advanced cirrhosis has been diagnosed with hepatic encephalopathy. The nurse expects to assess for: A. Malaise B. Stomatitis C. Hand tremors D. Weight loss

C Hepatic encephalopathy results from the accumulation of neurotoxins in the blood, therefore the nurse wants to assess for signs of neurological involvement. Flapping of the hands (asterixis), changes in mentation, agitation, and confusion are common. These clients typically have ascites and edema so experience weight gain. Malaise and stomatitis are not related to neurological involvement.

Nurse Joy is preparing to administer medication through a nasogastric tube that is connected to suction. To administer the medication, the nurse would: A. Position the client supine to assist in medication absorption B. Aspirate the nasogastric tube after medication administration to maintain patency C. Clamp the nasogastric tube for 30 minutes following administration of the medication D. Change the suction setting to low intermittent suction for 30 minutes after medication administration

C If a client has a nasogastric tube connected to suction, the nurse should wait up to 30 minutes before reconnecting the tube to the suction apparatus to allow adequate time for medication absorption. Aspirating the nasogastric tube will remove the medication just administered. Low intermittent suction also will remove the medication just administered. The client should not be placed in the supine position because of the risk for aspiration.

The nurse is performing a colostomy irrigation on a client. During the irrigation, a client begins to complain of abdominal cramps. Which of the following is the most appropriate nursing action? A. Notify the physician B. Increase the height of the irrigation C. Stop the irrigation temporarily D. Medicate with dilaudid and resume the irrigation

C If cramping occurs during a colostomy irrigation, the irrigation flow is stopped temporarily and the client is allowed to rest. Cramping may occur from an infusion that is too rapid or is causing too much pressure. Increasing the height of the irrigation will cause further discomfort. The physician does not need to be notified. Medicating the client for pain is not the most appropriate action

When administering sucralfate (Carafate) to a patient with a nasogastric tube, it is important to: A. Crush the tablet into a fine powder before mixing with water B. Administer with a bolus tube feeding C. Allow the tablet to dissolve in water before administering D. Administer with an antacid for maximum benefit

C It is important to give sucralfate on an empty stomach so that it may dissolve and form a protective barrier over the gastric mucosa. The tablet form will not dissolve in water when crushed; it must be left whole and allowed to dissolve. Crushing the medication so that it will not dissolve could lead to clogging of the nasogastric tube and decreased effectiveness of the drug.

A client has been taking aluminum hydroxide 30 mL six times per day at home to treat his peptic ulcer. He tells the nurse that he has been unable to have a bowel movement for 3 days. Based on this information, the nurse would determine that which of the following is the most likely cause of the client's constipation? A. The client has not been including enough fiber in his diet B. The client needs to increase his daily exercise C. The client is experiencing a side effect of the aluminum hydroxide. D. The client has developed a gastrointestinal obstruction

C It is most likely that the client is experiencing a side effect of the antacid. Antacids with aluminum salt products, such as aluminum hydroxide, form insoluble salts in the body. These precipitate and accumulate in the intestines, causing constipation. Increasing dietary fiber intake or daily exercise may be a beneficial lifestyle change for the client but is not likely to relieve the constipation caused by the aluminum hydroxide. Constipation, in isolation from other symptoms, is not a sign of bowel obstruction.

Which of the following interventions should be included in the medical management of Crohn's disease? A. Increasing oral intake of fiber B. Administering laxatives C. Using long-term steroid therapy D. Increasing physical activity

C Management of Crohn's disease may include long-term steroid therapy to reduce the inflammation associated with the deeper layers of the bowel wall. Other management focuses on bowel rest (not increasing oral intake) and reducing diarrhea with medications (not giving laxatives). The pain associated with Crohn's disease may require bed rest, not an increase in physical activity.

The nurse is reviewing the physician's orders written for a male client admitted to the hospital with acute pancreatitis. Which physician order should the nurse question if noted on the client's chart? A. NPO status B. Nasogastric tube inserted C. Morphine sulfate for pain D. An anticholinergic medication

C Meperidine (Demerol) rather than morphine sulfate is the medication of choice to treat pain because morphine sulfate can cause spasms in the sphincter of Oddi. Options A, B, and D are appropriate interventions for the client with acute pancreatitis.

After a subtotal gastrectomy, care of the client's nasogastric tube and drainage system should include which of the following nursing interventions? A. Irrigate the tube with 30 ml of sterile water every hour, if needed B. Reposition the tube if it is not draining well C. Monitor the client for N/V, and abdominal distention D. Turn the machine to high suction of the drainage is sluggish on low suction

C Nausea, vomiting, or abdominal distention indicated that gas and secretions are accumulating within the gastric pouch due to impaired peristalsis or edema at the operative site and may indicate that the drainage system is not working properly. Saline solution is used to irrigate nasogastric tubes. Hypotonic solutions such as water increase electrolyte loss. In addition, a physician's order is needed to irrigate the NG tube, because this procedure could disrupt the suture line. After gastric surgery, only the surgeon repositions the NG tube because of the danger of rupturing or dislodging the suture line. The amount of suction varies with the type of tube used and is ordered by the physician. High suction may create too much tension on the gastric suture line.

A client is to take one daily dose of ranitidine (Zantac) at home to treat her peptic ulcer. The nurse knows that the client understands proper drug administration of ranitidine when she says that she will take the drug at which of the following times? A. Before meals B. With meals C. At bedtime D. When pain occurs

C Ranitidine blocks secretion of hydrochloric acid. Clients who take only one daily dose of ranitidine are usually advised to take it at bedtime to inhibit nocturnal secretion of acid. Clients who take the drug twice a day are advised to take it in the morning and at bedtime.

The nurse is caring for a hospitalized female client with a diagnosis of ulcerative colitis. Which finding, if noted on assessment of the client, would the nurse report to the physician? A. Hypotension B. Bloody diarrhea C. Rebound tenderness D. A hemoglobin level of 12 mg/dL

C Rebound tenderness may indicate peritonitis. Bloody diarrhea is expected to occur in ulcerative colitis. Because of the blood loss, the client may be hypotensive and the hemoglobin level may be lower than normal. Signs of peritonitis must be reported to the physician.

A client with ulcerative colitis has an order to begin salicylate medication to reduce inflammation. The nurse instructs the client to take the medication: A. 30 minutes before meals B. On an empty stomach C. After meals D. On arising

C Salicylate compounds act by inhibiting prostaglandin synthesis and reducing inflammation. The nurse teaches the client to take the medication with a full glass of water and to increase fluid intake throughout the day. This medication needs to be taken after meals to reduce GI irritation.

Sucralfate (Carafate) achieves a therapeutic effect by: A. Neutralizing gastric acid B. Enhancing gastric absorption C. Forming a protective barrier around gastric mucosa D. Inhibiting gastric acid secretion

C Sucralfate has a local effect only on the gastric mucosa. It forms a paste-like substance in the stomach, which adheres to the gastric lining, protecting against adverse effects related to gastric acid. It also stimulates healing of any ulcerated areas of the gastric mucosa.

The nurse is performing an abdominal assessment and inspects the skin of the abdomen. The nurse performs which assessment technique next? A. Palpates the abdomen for size B. Palpates the liver at the right rib margin C. Listens to bowel sounds in all four quadrants D. Percusses the right lower abdominal quadrant

C The appropriate sequence for abdominal examination is inspection, auscultation, percussion, and palpation. Auscultation is performed after inspection to ensure that the motility of the bowel and bowel sounds are not altered by percussion or palpation. Therefore, after inspecting the skin on the abdomen, the nurse should listen for bowel sounds.

The client with a duodenal ulcer may exhibit which of the following findings on assessment? A. Hematemesis B. Malnourishment C. Melena D. Pain with eating

C The client with a duodenal ulcer may have bleeding at the ulcer site, which shows up as melena (black tarry poop). The other findings are consistent with a gastric ulcer.

During the assessment of a client's mouth, the nurse notes the absence of saliva. The client is also complaining of pain near the area of the ear. The client has been NPO for several days because of the insertion of a NG tube. Based on these findings, the nurse suspects that the client is developing which of the following mouth conditions? A. Stomatitis B. Oral candidiasis C. Parotitis D. Gingivitis

C The lack of saliva, pain near the area of the ear, and the prolonged NPO status of the client should lead the nurse to suspect the development of parotitis, or inflammation of the parotid gland. Parotitis usually develops in cases of dehydration combined with poor oral hygiene or when clients have been NPO for an extended period. Preventative measures include the use of sugarless hard candy or gum to stimulate saliva production, adequate hydration, and frequent mouth care. Stomatitis (inflammation of the mouth) produces excessive salivation and a sore mouth.

A client with suspected gastric cancer undergoes an endoscopy of the stomach. Which of the following assessments made after the procedure would indicate the development of a potential complication? A. The client complains of a sore throat B. The client displays signs of sedation C. The client experiences a sudden increase in temperature D. The client demonstrates a lack of appetite

C The most likely complication of an endoscopic procedure is perforation. A sudden temperature spike with 1 to 2 hours after the procedure is indicative of a perforation and should be reported immediately to the physician. A sore throat is to be anticipated after an endoscopy. Clients are given sedatives during the procedure, so it is expected that they will display signs of sedation after the procedure is completed. A lack of appetite could be the result of many factors, including the disease process.

Mucosal barrier fortifiers are used in peptic ulcer disease management for which of the following indications? A. To inhibit mucus production B. To neutralize acid production C. To stimulate mucus production D. To stimulate hydrogen ion diffusion back into the mucosa

C The mucosal barrier fortifiers stimulate mucus production and prevent hydrogen ion diffusion back into the mucosa, resulting in accelerated ulcer healing. Antacids neutralize acid production.

Nurse Berlinda is assigned to a 41-year-old client who has a diagnosis of chronic pancreatitis. The nurse reviews the laboratory result, anticipating a laboratory report that indicates a serum amylase level of: A. 45 units/L B. 100 units/L C. 300 units/L D. 500 units/L

C The normal serum amylase level is 25 to 151 units/L. With chronic cases of pancreatitis, the rise in serum amylase levels usually does not exceed three times the normal value. In acute pancreatitis, the value may exceed five times the normal value. Options A and B are within normal limits. Option D is an extremely elevated level seen in acute pancreatitis.

Which of the following medications is most effective for treating the pain associated with irritable bowel disease? A. Acetaminophen B. Opiates C. Steroids D. Stool softeners

C The pain with irritable bowel disease is caused by inflammation, which steroids can reduce. Stool softeners aren't necessary. Acetaminophen has little effect on the pain, and opiate narcotics won't treat its underlying cause

Which area of the alimentary canal is the most common location for Crohn's disease? A. Ascending colon B. Descending colon C. Sigmoid colon D. Terminal ileum

D Studies have shown that the terminal ileum is the most common site for recurrence in clients with Crohn's disease. The other areas may be involved but aren't as common.

The client being treated for esophageal varices has a Sengstaken-Blakemore tube inserted to control the bleeding. The most important assessment is for the nurse to: A. Check that the hemostat is on the bedside B. Monitor IV fluids for the shift C. Regularly assess respiratory status D. Check that the balloon is deflated on a regular basis

C The respiratory system can become occluded if the balloon slips and moves up the esophagus, putting pressure on the trachea. This would result in respiratory distress and should be assessed frequently. Scissors should be kept at the bedside to cut the tube if distress occurs. This is a safety intervention.

A 30-year old client experiences weight loss, abdominal distention, crampy abdominal pain, and intermittent diarrhea after birth of her 2nd child. Diagnostic tests reveal gluten-induced enteropathy. Which foods must she eliminate from her diet permanently? A. Milk and dairy products B. Protein-containing foods C. Cereal grains (except rice and corn) D. Carbohydrates

C To manage gluten-induced enteropathy, the client must eliminate gluten, which means avoiding all cereal grains except for rice and corn. In initial disease management, clients eat a high calorie, high-protein diet with mineral and vitamin supplements to help normalize nutritional status.

Pierre who is diagnosed with acute pancreatitis is under the care of Nurse Bryan. Which intervention should the nurse include in the care plan for the client? A. Administration of vasopressin and insertion of a balloon tamponade B. Preparation for a paracentesis and administration of diuretics C. Maintenance of nothing-by-mouth status and insertion of nasogastric (NG) tube with low intermittent suction D. Dietary plan of a low-fat diet and increased fluid intake to 2,000 ml/day

C With acute pancreatitis, the client is kept on nothing-by-mouth status to inhibit pancreatic stimulation and secretion of pancreatic enzymes. NG intubation with low intermittent suction is used to relieve nausea and vomiting, decrease painful abdominal distention, and remove hydrochloric acid. Vasopressin would be appropriate for a client diagnosed with bleeding esophageal varices. Paracentesis and diuretics would be appropriate for a client diagnosed with portal hypertension and ascites. A low-fat diet and increased fluid intake would further aggravate the pancreatitis.

During the treatment of the patient with bleeding esophageal varices, it is most important that the nurse a. prepare the patient for immediate portal shunting surgery b. perform guaiac testing on all stools to detect occult blood c. maintain the patient's airway and prevent aspiration of blood d. monitor for the cardiac effects of IV vasopressin and nitroglycerin

C- Bleeding esophageal varices are a medical emergency. During an episode of bleeding, management of the airway and prevention of aspiration of blood are critical factors. Occult blood as well as fresh blood from the GI tract would be expected and is not tested. Vasopressin causes vasoconstriction, decreased HR, and decreased coronary blood flow; nitroglycerin is given with the vasopressin to counter these side effects. Portal shunting surgery is performed for esophageal varices but not during an acute hemorrhage

During the incubation period of viral hepatitis, the nurse would expect the patient to report a. pruritus and malaise b. dark urine and easy fatigability c. anorexia and right upper quadrant discomfort d. constipation or diarrhea with light colored stools

C- Incubation symptoms occur before the onset of jaundice and include a variety of GI symptoms as well as discomfort and heaviness in the upper right quadrant of the abdomen. Pruritus, dark urine, and light colored stools occur with the onset of jaundice in the acute phase.

The nurse is planning to teach a client with GERD about substances to avoid. Which items should the nurse include on this list? Select all that apply. Coffee Chocolate Peppermint Nonfat milk Fried chicken Scrambled eggs

Coffee, chocolate, peppermint, fried chicken

A patient has been admitted to the medical unit after several days of watery diarrhea related to Crohn's disease. The healthcare provider recognizes which of the following symptoms as most concerning? Please choose from one of the following options. A. Right upper quadrant pain B. Elevated hematocrit C. Elevated leukocytes D. Palpitations

D

During an acute exacerbation of inflammatory bowel disease, a patient is to receive total parenteral nutrition (TPN) and lipids. Which of these interventions is the priority when caring for this patient? Please choose from one of the following options. A. Monitor urine specific gravity every shift B. Change the administration set every 72 hours C. Infuse the solution in a large peripheral vein D. Monitor the patient's blood glucose per protocol

D

One of your patients is receiving digitalis orally and is also to receive an antacid at the same time. Your most appropriate action, based on the pharmacokinetics of antacids, is to: A. Delay the digitalis for 1 to 2 hours until the antacid is absorbed B. Give the antacid at least 2 to 4 hours before administering the digitalis C. Administer both medications as ordered and document in nurse's notes D. Contact the physician regarding the drug interaction and request a change in the time of dosing of the drugs

D

If a client had irritable bowel syndrome, which of the following diagnostic tests would determine if the diagnosis is Crohn's disease or ulcerative colitis? A. Abdominal computed tomography (CT) scan B. Abdominal x-ray C. Barium swallow D. Colonoscopy with biopsy

D A colonoscopy with biopsy can be performed to determine the state of the colon's mucosal layers, presence of ulcerations, and level of cytologic development. An abdominal x-ray or CT scan wouldn't provide the cytologic information necessary to diagnose which disease it is. A barium swallow doesn't involve the intestine.

A client with a peptic ulcer is scheduled for a vagotomy. The client asks the nurse about the purpose of this procedure. The nurse tells the client that the procedure: A. Decreases food absorption in the stomach B. Heals the gastric mucosa C. Halts stress reactions D. Reduces the stimulus to acid secretions

D A vagotomy, or cutting the vagus nerve, is done to eliminate parasympathetic stimulation of gastric secretion.

Which clinical manifestation would the nurse expect a client diagnosed with acute cholecystitis to exhibit? A. Jaundice, dark urine, and steatorrhea B. Acute right lower quadrant (RLQ) pain, diarrhea, and dehydration C. Ecchymosis petechiae, and coffee-ground emesis D. Nausea, vomiting, and anorexia

D Acute cholecystitis is an acute inflammation of the gallbladder commonly manifested by the following: anorexia, nausea, and vomiting; biliary colic; tenderness and rigidity the right upper quadrant (RUQ) elicited on palpation (e.g., Murphy's sign); fever; fat intolerance; and signs and symptoms of jaundice. Ecchymosis, petechiae, and coffee-ground emesis are clinical manifestations of esophageal bleeding. The coffee-ground appearance indicates old bleeding. Jaundice, dark urine, and steatorrhea are clinical manifestations of the icteric phase of hepatitis.

Which rationale supports explaining the placement of an esophageal tamponade tube in a client who is hemorrhaging? A. Allowing the client to help insert the tube B. Beginning teaching for home care C. Maintaining the client's level of anxiety and alertness D. Obtaining cooperation and reducing fear

D An esophageal tamponade tube would be inserted in critical situations. Typically, the client is fearful and highly anxious. The nurse therefore explains about the placement to help obtain the client's cooperation and reduce his fear. This type of tube is used only short term and is not indicated for home use. The tube is large and uncomfortable. The client would not be helping to insert the tube. A client's anxiety should be decreased, not maintained, and depending on the degree of hemorrhage, the client may not be alert.

A client is taking an antacid for treatment of a peptic ulcer. Which of the following statements best indicates that the client understands how to correctly take the antacid? A. "I should take my antacid before I take my other medications." B. "I need to decrease my intake of fluids so that I don't dilute the effects of my antacid." C. "My antacid will be most effective if I take it whenever I experience stomach pains." D. "It is best for me to take my antacid 1 to 3 hours after meals."

D Antacids are most effective if taken 1 to 3 hours after meals and at bedtime. When an antacid is taken on an empty stomach, the duration of the drug's action is greatly decreased. Taking antacids 1 to 3 hours after a meal lengthens the duration of action, thus increasing the therapeutic action of the drug. Antacids should be administered about 2 hours after other medications to decrease the chance of drug interactions. It is not necessary to decrease fluid intake when taking antacids.

Which of the following laboratory results would be expected in a client with peritonitis? A. Partial thromboplastin time above 100 seconds B. Hemoglobin level below 10 mg/dL C. Potassium level above 5.5 mEq/L D. White blood cell count above 15,000

D Because of infection, the client's WBC count will be elevated. A hemoglobin level below 10 mg/dl may occur from hemorrhage. A PT time longer than 100 seconds may suggest disseminated intravascular coagulation, a serious complication of septic shock. A potassium level above 5.5 mEq/L may indicate renal failure.

Colon cancer is most closely associated with which of the following conditions? A. Appendicitis B. Hemorrhoids C. Hiatal hernia D. Ulcerative colitis

D Chronic ulcerative colitis, granulomas, and familial polposis seem to increase a person's chance of developing colon cancer. The other conditions listed have no known effect on colon cancer risk.

The client with GERD complains of a chronic cough. The nurse understands that in a client with GERD this symptom may be indicative of which of the following conditions? A. Development of laryngeal cancer B. Irritation of the esophagus C. Esophageal scar tissue formation D. Aspiration of gastric contents

D Clients with GERD can develop pulmonary symptoms such as coughing, wheezing, and dyspnea that are caused by the aspiration of gastric contents. GERD does not predispose the client to the development of laryngeal cancer. Irritation of the esophagus and esophageal scar tissue formation can develop as a result of GERD. However, GERD is more likely to cause painful and difficult swallowing.

A client being treated for chronic cholecystitis should be given which of the following instructions? A. Increase rest B. Avoid antacids C. Increase protein in diet D. Use anticholinergics as prescribed

D Conservative therapy for chronic cholecystitis includes weight reduction by increasing physical activity, a low-fat diet, antacid use to treat dyspepsia, and anticholinergic use to relax smooth muscles and reduce ductal tone and spasm, thereby reducing pain.

Which of the following symptoms may be exhibited by a client with Crohn's disease? A. Bloody diarrhea B. Narrow stools C. N/V D. Steatorrhea

D Steatorrhea from malaborption can occur with Crohn's disease. N/V, and bloody diarrhea are symptoms of ulcerative colitis. Narrow stools are associated with diverticular disease.

Crohn's disease can be described as a chronic relapsing disease. Which of the following areas in the GI system may be involved with this disease? A. The entire length of the large colon B. Only the sigmoid area C. The entire large colon through the layers of mucosa and submucosa D. The small intestine and colon; affecting the entire thickness of the bowel

D Crohn's disease can involve any segment of the small intestine, the colon, or both, affecting the entire thickness of the bowel. Answers 1 and 3 describe ulcerative colitis, answer 2 is too specific and therefore, not likely.

A nurse is inserting a nasogastric tube in an adult male client. During the procedure, the client begins to cough and has difficulty breathing. Which of the following is the appropriate nursing action? A. Quickly insert the tube B. Notify the physician immediately C. Remove the tube and reinsert when the respiratory distress subsides D. Pull back on the tube and wait until the respiratory distress subsides

D During the insertion of a nasogastric tube, if the client experiences difficulty breathing or any respiratory distress, withdraw the tube slightly, stop the tube advancement, and wait until the distress subsides. Options B and C are unnecessary. Quickly inserting the tube is not an appropriate action because, in this situation, it may be likely that the tube has entered the bronchus.

When teaching an elderly client how to prevent constipation, which of the following instructions should the nurse include? A. "Drink 6 glasses of fluid each day." B. "Avoid grain products and nuts." C. "Add at least 4 grams of bran to your cereal each morning." D. "Be sure to get regular exercise."

D Exercise helps prevent constipation. Fluids and dietary fiber promote normal bowel function. The client should drink eight to ten glasses of fluid each day. Although adding bran to cereal helps prevent constipation by increasing dietary fiber, the client should start with a small amount and gradually increase the amount as tolerated to a maximum of 2 grams a day.

A client presents to the emergency room, reporting that he has been vomiting every 30 to 40 minutes for the past 8 hours. Frequent vomiting puts him at risk for which of the following? A. Metabolic acidosis with hyperkalemia B. Metabolic acidosis with hypokalemia C. Metabolic alkalosis with hyperkalemia D. Metabolic alkalosis with hypokalemia

D Gastric acid contains large amounts of potassium, chloride, and hydrogen ions. Excessive loss of these substances, such as from vomiting, can lead to metabolic alkalosis and hypokalemia.

The nurse would question an order for which type of antacid in patients with chronic renal failure? A. Aluminum-containing antacids B. Calcium-containing antacids C. Magnesium-containing antacids D. All of the above

D Magnesium-containing antacids can cause hypermagnesemia in patients with chronic renal failure. Aluminum-containing antacids may be used as a phosphate binder in patients with chronic renal failure. Calcium-containing antacids are also appropriate because these patients may be hypocalcemic.

The nurse is reviewing the physician's orders written for a client admitted with acute pancreatitis. Which physician order would the nurse question if noted on the client's chart? A. NPO status B. Insert a nasogastric tube C. An anticholinergic medication D. Morphine for pain

D Meperidine (Demerol) rather than morphine is the medication of choice because morphine can cause spasm in the sphincter of Oddi.

When counseling a client in ways to prevent cholecystitis, which of the following guidelines is most important? A. Eat a low-protein diet B. Eat a low-fat, low-cholesterol diet C. Limit exercise to 10 minutes/day D. Keep weight proportionate to height

D Obesity is a known cause of gallstones, and maintaining a recommended weight will help protect against gallstones. Excessive dietary intake of cholesterol is associated with the development of gallstones in many people. Dietary protein isn't implicated in cholecystitis. Liquid protein and low-calorie diets (with rapid weight loss of more than 5 lb [2.3kg] per week) are implicated as the cause of some cases of cholecystitis. Regular exercise (30 minutes/three times a week) may help reduce weight and improve fat metabolism. Reducing stress may reduce bile production, which may also indirectly decrease the chances of developing cholecystitis.

The client with peptic ulcer disease is scheduled for a pyloroplasty. The client asks the nurse about the procedure. The nurse plans to respond knowing that a pyloroplasty involves: A. Cutting the vagus nerve B. Removing the distal portion of the stomach C. Removal of the ulcer and a large portion of the cells that produce hydrochloric acid D. An incision and resuturing of the pylorus to relax the muscle and enlarge the opening from the stomach to the duodenum

D Option 4 describes the procedure for a pyloroplasty. A vagotomy involves cutting the vagus nerve. A subtotal gastrectomy involves removing the distal portion of the stomach. A Billroth II procedure involves removal of the ulcer and a large portion of the tissue that produces hydrochloric acid.

The pain of a duodenal ulcer can be distinguished from that of a gastric ulcer by which of the following characteristics? A. Early satiety B. Pain on eating C. Dull upper epigastric pain D. Pain on empty stomach

D Pain on empty stomach is relieved by taking foods or antacids. The other symptoms are those of a gastric ulcer.

The nurse is monitoring a female client with a diagnosis of peptic ulcer. Which assessment findings would most likely indicate perforation of the ulcer? A. Bradycardia B. Numbness in the legs C. Nausea and vomiting D. A rigid, board-like abdomen

D Perforation of an ulcer is a surgical emergency and is characterized by sudden, sharp, intolerable severe pain beginning in the midepigastric area and spreading over the abdomen, which becomes rigid and board-like. Nausea and vomiting may occur. Tachycardia may occur as hypovolemic shock develops. Numbness in the legs is not an associated finding.

Surgical management of ulcerative colitis may be performed to treat which of the following complications? A. Gastritis B. Bowel herniation C. Bowel outpouching D. Bowel perforation

D Perforation, obstruction, hemorrhage, and toxic megacolon are common complications of ulcerative colitis that may require surgery. Herniation and gastritis aren't associated with irritable bowel diseases, and outpouching of the bowel is diverticulosis.

The nurse is caring for a hospitalized client with a diagnosis of ulcerative colitis. Which finding, if noted on assessment of the client, would the nurse report to the physician? A. Bloody diarrhea B. Hypotension C. A hemoglobin of 12 mg/dL D. Rebound tenderness

D Rebound tenderness may indicate peritonitis. Blood diarrhea is expected to occur in ulcerative colitis. Because of the blood loss, the client may be hypotensive and the hemoglobin level may be lower than normal. Signs of peritonitis must be reported to the physician.

Which of the following diagnostic tests should be performed annually over age 50 to screen for colon cancer? A. Abdominal CT scan B. Abdominal x-ray C. Colonoscopy D. Fecal occult blood test

D Surface blood vessels of polyps and cancers are fragile and often bleed with the passage of stools. Abdominal x-ray and CT scan can help establish tumor size and metastasis. A colonoscopy can help locate a tumor as well as polyps, which can be removed before they become malignant.

Which of the following tests can be used to diagnose ulcers? A. Abdominal x-ray B. Barium swallow C. Computed tomography (CT) scan D. Esophagogastroduodenoscopy (EGD)

D The EGD can visualize the entire upper GI tract as well as allow for tissue specimens and electrocautery if needed. The barium swallow could locate a gastric ulcer. A CT scan and an abdominal x-ray aren't useful in the diagnosis of an ulcer.

A client is admitted to the hospital after vomiting bright red blood and is diagnosed with a bleeding duodenal ulcer. The client develops a sudden, sharp pain in the mid epigastric area along with a rigid, board-like abdomen. These clinical manifestations most likely indicate which of the following? A. An intestinal obstruction has developed B. Additional ulcers have developed C. The esophagus has become inflamed D. The ulcer has perforated

D The body reacts to perforation of an ulcer by immobilizing the area as much as possible. This results in boardlike muscle rigidity, usually with extreme pain. Perforation is a medical emergency requiring immediate surgical intervention because peritonitis develops quickly after perforation. An intestinal obstruction would not cause midepigastric pain. Esophageal inflammation or the development of additional ulcers would not cause a rigid, boardlike abdomen.

The client who has undergone creation of a colostomy has a nursing diagnosis of Disturbed body image. The nurse would evaluate that the client is making the most significant progress toward identified goals if the client: A. Watches the nurse empty the colostomy bag B. Looks at the ostomy site C. Reads the ostomy product literature D. Practices cutting the ostomy appliance

D The client is expected to have a body image disturbance after colostomy. The client progresses through normal grieving stages to adjust to this change. The client demonstrates the greatest deal of acceptance when the client participates in the actual colostomy care. Each of the incorrect options represents an interest in colostomy care but is a passive activity. The correct option shows the client is participating in self-care.

Which of the following tasks should be included in the immediate postoperative management of a client who has undergone gastric resection? A. Monitoring gastric pH to detect complications B. Assessing for bowel sounds C. Providing nutritional support D. Monitoring for symptoms of hemorrhage

D The client should be monitored closely for signs and symptoms of hemorrhage, such as bright red blood in the nasogastric tube suction, tachycardia, or a drop in blood pressure. Gastric pH may be monitored to evaluate the need for histamine-2 receptor antagonists. Bowel sounds may not return for up to 72 hours postoperatively. Nutritional needs should be addressed soon after surgery.

Jordin is a client with jaundice who is experiencing pruritus. Which nursing intervention would be included in the care plan for the client? A. Administering vitamin K subcutaneously B. Applying pressure when giving I.M. injections C. Decreasing the client's dietary protein intake D. Keeping the client's fingernails short and smooth

D The client with pruritus experiences itching, which may lead to skin breakdown and possibly infection from scratching. Keeping his fingernails short and smooth helps prevent skin breakdown and infection from scratching. Applying pressure when giving I.M. injections and administering vitamin K subcutaneously are important if the client develops bleeding problems. Decreasing the client's dietary intake is appropriate if the client's ammonia levels are increased.

Which of the following nursing interventions should be implemented to manage a client with appendicitis? A. Assessing for pain B. Encouraging oral intake of clear fluids C. Providing discharge teaching D. Assessing for symptoms of peritonitis

D The focus of care is to assess for peritonitis, or inflammation of the peritoneal cavity. Peritonitis is most commonly caused by appendix rupture and invasion of bacteria, which could be lethal. The client with appendicitis will have pain that should be controlled with analgesia. The nurse should discourage oral intake in preparation of surgery. Discharge teaching is important; however, in the acute phase, management should focus on minimizing preoperative complications and recognizing when such may be occurring.

Polyethylene glycol-electrolyte solution (GoLYTELY) is prescribed for the female client scheduled for a colonoscopy. The client begins to experience diarrhea following administration of the solution. What action by the nurse is appropriate? A. Start an IV infusion B. Administer an enema C. Cancel the diagnostic test D. Explain that diarrhea is expected

D The solution GoLYTELY is a bowel evacuant used to prepare a client for a colonoscopy by cleansing the bowel. The solution is expected to cause a mild diarrhea and will clear the bowel in 4 to 5 hours. Options A, B, and C are inappropriate actions

Which of the following associated disorders may a client with ulcerative colitis exhibit? A. Gallstones B. Hydronephrosis C. Nephrolithiasis D. Toxic megacolon

D Toxic megacolon is extreme dilation of a segment of the diseased colon caused by paralysis of the colon, resulting in complete obstruction. This disorder is associated with both Crohn's disease and ulcerative colitis. The other disorders are more commonly associated with Crohn's disease.

Which of the following mechanisms can facilitate the development of diverticulosis into diverticulitis? A. Treating constipation with chronic laxative use, leading to dependence on laxatives B. Chronic constipation causing an obstruction, reducing forward flow of intestinal contents C. Herniation of the intestinal mucosa, rupturing the wall of the intestine D. Undigested food blocking the diverticulum, predisposing the area to bacterial invasion

D Undigested food can block the diverticulum, decreasing blood supply to the area and predisposing the area to invasion of bacteria. Chronic laxative use is a common problem in elderly clients, but it doesn't cause diverticulitis. Chronic constipation can cause an obstruction—not diverticulitis. Herniation of the intestinal mucosa causes an intestinal perforation.

A nurse is preparing to care for a female client with esophageal varices who has just has a Sengstaken-Blakemore tube inserted. The nurse gathers supplies, knowing that which of the following items must be kept at the bedside at all times? A. An obturator B. .Kelly clamp C. An irrigation set D. A pair of scissors

D When the client has a Sengstaken-Blakemore tube, a pair of scissors must be kept at the client's bedside at all times. The client needs to be observed for sudden respiratory distress, which occurs if the gastric balloon ruptures and the entire tube moves upward. If this occurs, the nurse immediately cuts all balloon lumens and removes the tube. An obturator and a Kelly clamp are kept at the bedside of a client with a tracheostomy. An irrigation set may be kept at the bedside, but it is not the priority item.

Which of the following conditions can cause a hiatal hernia? A. Increased intrathoracic pressure B. Weakness of the esophageal muscle C. Increased esophageal muscle pressure D. Weakness of the diaphragmic muscle

D A hiatal hernia is caused by weakness of the diaphragmic muscle and increased intra-abdominal—not intrathoracic—pressure. This weakness allows the stomach to slide into the esophagus. The esophageal supports weaken, but esophageal muscle weakness or increased esophageal muscle pressure isn't a factor in hiatal hernia.

The nurse is instructing a patient with chronic pancreatitis on measures to prevent further attacks. What information should be provided (select all that apply) a. avoid nicotine b. eat bland foods c. observe stools for steatorrhea d. eat high fat, low protein, high carbohydrate meals e. take prescribed pancreatic enzymes immediately following meals

a. avoid nicotine b. eat bland foods c. observe stools for steatorrhea Measures to prevent attacks of pancreatitis are those that decrease stimulation of the pancreas. lower fat intake, and foods that are irritating (eat bland), higher carbs are less stimulating, avoid alcohol and nicotine, monitor for steatorrhea (fat in feces). Pancreatic enzymes should be taken with meals, not after.

The patient with suspected gallbladder disease is scheduled for an ultrasound of the gallbladder. The nurse explains to the patient that this test a. is noninvasive and is a very reliable method of detecting gallstones b. is used only when other tests cannot be used because of allergy to contrast media c. is an adjunct to liver function tests to determine whether the gallbladder is inflamed d. will outline the gallbladder and the ductal system to enable visualization of stones

a. is noninvasive and is a very reliable method of detecting gallstones Ultrasonography is 90-95% accurate in detecting gallstones, and is noninvasive. Liver function tests will be elevated if there is damage to the liver, not with gallbladder.

A risk factor associated with cancer of the pancreas is a. alcohol intake b. cigarette smoking c. exposure to asbestos d. increased dietary intake of milk and milk products

b. cigarette smoking

A patient with acute pancreatitis has a nursing diagnosis of pain related to distention of pancreas and peritoneal irritation. In addition to effective use of analgesics, the nurse should a. provider diversional activities to distract the patient from pain b. provide small frequent meals to increase the patient's tolerance of food c. position the patient on the side with the head of the bed elevated 45 degrees for pain relief d. ambulate the patient every 3 to 4 hours to increase circulation and decrease abdominal congestion

c. position the patient on the side with the head of the bed elevated 45 degrees for pain relief

The nurse explains to the pt with GERD that this disorder: a) results in acid erosion and ulceration of the esophagus caused by frequent vomiting. b) will require surgical wrapping or repair of the pyloric sphincter to control the symptoms. c) is the protrusion of a portion of the stomach into the esophagus through an opening in the diaphragm d) often involves relaxation of the LES, allowing stomach contents to back up into the esophagus.

d


Conjuntos de estudio relacionados

NCCT phlebotomy interactive review

View Set

Fed Gov Chapter 8: Political Parties, Candidates, and Campaigns

View Set

N144 - Exam 2 - Practice Questions

View Set

M7- Dimensions and Strategies of Development

View Set

Prep U Chapter 34: Assessment and Management of Patients with Inflammatory Rheumatic Disorders

View Set